topicos de eletromagnetismo i

86
FIW 591 Tópicos de Eletro Objetivos: Introduzir a teoria eletroma particularmente suas aplicações. Ementa: Primeira parte (P1): Análise vetorial (capítulo 1 - 5 au Eletrostática (capítulo 2 – 5 aula Segunda parte (P2): Magnetostática (capítulo 5 – 4 a Eletrodinâmica (capítulo 7 – 4 au Terceira parte (P3): Leis de conservação (capítulo 8 Ondas eletromagnéticas (capítul Avaliação: 3 provas (Pi, i= 1,2,3) + listas em onde L i é a média entre as 75 daquele período correspondent segunda chamada (S) e um exam prova será atribuída uma nota (N = 0,7*P i + 0,3*L i Bibliografia: Livro texto: G Referências a REITZ, J.R, Eletromagné Kleber Dau UEPG, 2004. Anita Mace A lista completa está disponível Dicas de sites: http://web.mit.edu/8.02t/www/ Não se pode ensinar alguma omagnetismo I Prof. Antônio Carlos Universidade Federal do Rio de Janeiro Instituto de Física Curso de Licenciatura em Física FIW 591 Tópicos de Eletromagnetismo I (http://www.if.ufrj.br/~toni/top_eletro.pdf) Prof. Antônio Carlos ([email protected]) agnética de Maxwell, explorando o seu aspec s. ulas) as) aulas) ulas) 2 aulas) lo 9 – 6 aulas) m sala de aula (L i ), 5% maiores notas te, uma prova de me final (E). A cada N i , i=1,2,3) onde N i Cálculo da Média (M): Presente às provas parciais: M = (N 1 + N 2 + N 3 )/3 Se M < 3,0, então reprovado com Se M > ou igual a 7,0, então apro igual à M Se 7,0 > M > ou igual a 3,0, entã Ausente em uma das provas Fará o exame final obrigatoriam calculado como anteriormente, substituindo a nota da prova não Pedidos de revisão: Os pedidos de revisão deverão forma escrita com informação porquê o aluno acredita que de mais pontos. (dizer somente “p questão tal” não é suficiente). GRIFFITHS, D.J., Eletrodinâmica, Pearson Education, T adicionais: , MILFORD, F.J., CHRISTY, R.W., Fundamento ética, Rio de Janeiro: Editora Campus, 1982. um Machado, Teoria do Eletromagnetismo, vols. 1 edo, Eletromagnetismo, Editora Guanabara. em: http://omnis.if.ufrj.br/~toni/top_eletro. /802TEAL3D/visualizations/ (animações superlegais) coisa a alguém, pode-se apenas auxiliar a descobr 1 cto matemático e m grau igual à M ovado com grau ão grau =(M + E)/2; mente. M será com E o realizada. ser submetidos na detalhada sobre o everia ter recebido por favor revise a Terceira Edição. os da Teoria 1,2 e 3, Editora rir por si mesmo”

Upload: buithien

Post on 07-Jan-2017

279 views

Category:

Documents


9 download

TRANSCRIPT

Page 1: Topicos de Eletromagnetismo I

FIW 591 – Tópicos de Eletromagnetismo I

Objetivos: Introduzir a teoria eletromagnética de Maxwell, explorando o seu aspecto matemático e particularmente suas aplicações.

Ementa: Primeira parte (P1): Análise vetorial (capítulo 1 - 5 aulas)Eletrostática (capítulo 2 – 5 aulas)Segunda parte (P2): Magnetostática (capítulo 5 – 4 aulas)Eletrodinâmica (capítulo 7 – 4 aulas)Terceira parte (P3): Leis de conservação (capítulo 8 –Ondas eletromagnéticas (capítulo 9 Avaliação: 3 provas (Pi, i= 1,2,3) + listas em sala de aula (Londe Li é a média entre as 75% maiores notas daquele período correspondente, uma prova de segunda chamada (S) e um exame final (E). A cada prova será atribuída uma nota (N= 0,7*Pi + 0,3*Li

Bibliografia:

Livro texto: GRIFFITHS, D.J., Referências adicionais:

•REITZ, J.R, MILFORD, F.J.Eletromagnética,

•Kleber Daum Machado, UEPG, 2004.

• Anita Macedo,

A lista completa está disponível em:

Dicas de sites:

http://web.mit.edu/8.02t/www/802TEAL3D/visualizations/

“Não se pode ensinar alguma coisa a alguém, pode

Tópicos de Eletromagnetismo I – Prof. Antônio Carlos

Universidade Federal do Rio de Janeiro Instituto de Física

Curso de Licenciatura em Física FIW 591 Tópicos de Eletromagnetismo I

(http://www.if.ufrj.br/~toni/top_eletro.pdf) Prof. Antônio Carlos ([email protected])

Introduzir a teoria eletromagnética de Maxwell, explorando o seu aspecto matemático e particularmente suas aplicações.

5 aulas) 5 aulas)

4 aulas) 4 aulas)

– 2 aulas) Ondas eletromagnéticas (capítulo 9 – 6 aulas)

listas em sala de aula (Li), é a média entre as 75% maiores notas

daquele período correspondente, uma prova de segunda chamada (S) e um exame final (E). A cada prova será atribuída uma nota (Ni, i=1,2,3) onde Ni

Cálculo da Média (M): Presente às provas parciais: M = (N1 + N2 + N3)/3 Se M < 3,0, então reprovado com grau igual à MSe M > ou igual a 7,0, então aprovado com grau igual à M Se 7,0 > M > ou igual a 3,0, então grau =(M + E)/2; Ausente em uma das provas Fará o exame final obrigatoriamente. M será calculado como anteriormente, com E substituindo a nota da prova não realizada.Pedidos de revisão: Os pedidos de revisão deverão ser submetidos na forma escrita com informação detalhada sobre o porquê o aluno acredita que demais pontos. (dizer somente “por favor revise a questão tal” não é suficiente).

GRIFFITHS, D.J., Eletrodinâmica, Pearson Education, Terceira Edição.

Referências adicionais:

REITZ, J.R, MILFORD, F.J., CHRISTY, R.W., Fundamentos da Teoria

Eletromagnética, Rio de Janeiro: Editora Campus, 1982.

Kleber Daum Machado, Teoria do Eletromagnetismo, vols. 1,2 e 3, Editora

Anita Macedo, Eletromagnetismo, Editora Guanabara.

disponível em: http://omnis.if.ufrj.br/~toni/top_eletro.

http://web.mit.edu/8.02t/www/802TEAL3D/visualizations/ (animações superlegais)

se pode ensinar alguma coisa a alguém, pode-se apenas auxiliar a descobrir por si mesmo”

1

Introduzir a teoria eletromagnética de Maxwell, explorando o seu aspecto matemático e

Se M < 3,0, então reprovado com grau igual à M Se M > ou igual a 7,0, então aprovado com grau

7,0 > M > ou igual a 3,0, então grau =(M + E)/2;

riamente. M será calculado como anteriormente, com E substituindo a nota da prova não realizada.

Os pedidos de revisão deverão ser submetidos na forma escrita com informação detalhada sobre o porquê o aluno acredita que deveria ter recebido mais pontos. (dizer somente “por favor revise a

, Pearson Education, Terceira Edição.

Fundamentos da Teoria

, vols. 1,2 e 3, Editora

se apenas auxiliar a descobrir por si mesmo”

Page 2: Topicos de Eletromagnetismo I

2 FIW 591 – Tópicos de Eletromagnetismo I – Prof. Antônio Carlos

O que é ensino interativo? Dr. Louis Abrahamson (tradução e adaptação livre)

A primeira coisa a entender sobre o ensino interativo é que não é algo novo ou misterioso. Se você é um professor e

faz perguntas em sala de aula, atribui e verifica a lição de casa, ou mantém discussões em classe ou em grupo,

então você já ensina de forma interativa. Basicamente, então, o ensino interativo trata-se apenas de dar aos alunos

algo para fazer, recebendo de volta o que eles têm feito, e depois assimilando, de modo que você possa decidir

sobre o melhor fazer a seguir.

Mas, quase todos os professores já fazem essas coisas, assim o que há de novo? Para responder a esta questão,

devemos pensar sobre o processo ensino aprendizagem. Nos últimos vinte anos, o campo da ciência cognitiva nos

ensinou muito sobre como as pessoas aprendem. Um princípio central que tem sido geralmente aceito é o de que

tudo o que aprendemos, nós "construimos" para nós mesmos. Isto é, qualquer agente externo é essencialmente

impotente para ter um efeito direto sobre o que aprendemos. Se o nosso cérebro não fazê-lo em si, - isto é, levar

em informação, procurar conexões, interpretar e dar sentido a ela, - nenhuma força externa terá qualquer efeito.

Isso não significa que o esforço tem que ser expressamente voluntário e consciente da nossa parte. Nosso cérebro

fornece-nos informações e opera continuamente em vários de níveis, dos quais apenas alguns são conscientemente

dirigidos. Mas, consciente ou não, a coisa importante a entender é que é o nosso cérebro que esta realizando o

processo de aprendizagem, e que este processo está apenas indiretamente relacionado com o professor e do

ensino.

Por exemplo, mesmo uma exposição lúcida e brilhante sobre um assunto por um professor em uma aula, pode

resultar numa aprendizagem limitada se os cérebros dos alunos não realizarem o trabalho necessário para

processá-la. Há várias causas possíveis para a aprendizagem dos alunos ficarem aquém das expectativas em tal

situação. Eles podem, não entender totalmente um conceito crucial sobre um determinado assunto e assim o

assunto seguitne torna-se ininteligível. Pode também estar faltando informação prévia ou não ter uma boa

compreensão do que foi visto antes. Consequentemente as estruturas conceituais sobre as quais se baseia a aula

ficam ausentes. Falta de interesse, de motivação, ou não querer realizar um esforço mental para acompanhar a

aula, de entender os argumentos, etc...

No entanto, qualquer que seja a causa, sem interagir com os alunos (no caso mais simples, fazendo perguntas), um

professor não tem como saber se o seu esforço para explicar o tema foi bem sucedido.

Isto leva-me ao primeiro (o que eu acredito que são) três razões distintas para o ensino interativo. É uma tentativa

para ver o que realmente existe no cérebro de seus alunos. Este é o aspecto "sumativo". Este é o aspecto mais fácil

de compreender e está bem descrito na literatura. Mas, ele está longe de ser a única perspectiva! A segunda razão

é "formativa", onde o professor tem como objetivo, através da tarefa atribuída, acessar o processamento mental

dos alunos. A intenção é que, conforme os alunos pensem nas questões necessárias para chegar à solução, a

construção mental resultante que é desenvolvida na cabeça do aluno irá possuir as propriedades que o professor

está tentando ensinar. Como Sócrates descobriu, uma boa pergunta pode realizar este resultado melhor do que,

apenas dizer a resposta.

O terceiro aspectro pode ser chamado de "motivacional". Aprender é um trabalho duro, e uma injeção de

motivação no momento certo pode fazer toda a diferença. Um fator de motivação fornecido pelo professor

interativo é a exigência de uma resposta a uma tarefa em sala de aula. Isso serve para sacudir o aluno para a ação,

para tirar o seu cérebro da preguiça, por assim dizer. Eventos adicionais mais sutis e agradáveis podem vir a seguir

aproveitando o impulso criado por esta explosão inicial. Um deles é um resultado das nossas tendências humano-

sociais. Quando os professores pedem aos alunos que trabalhem juntos em pequenos grupos para resolver um

problema, uma discussão se segue que não serve apenas em si mesma para construir estruturas de conhecimento

mais robustas, mas também para motivar. A antecipação de feedback imediato na forma de reação de seus pares,

ou do professor é um elemento motivador muito forte. Se não for constrangedor ou ameaçador, os alunos desejam

saber se seu entendimento está progredindo ou apenas à deriva. Saber que eles não estão autorizados a vagar

longe demais fora da pista proporciona uma enorme energia para continuar

Page 3: Topicos de Eletromagnetismo I

3 FIW 591 – Tópicos de Eletromagnetismo I – Prof. Antônio Carlos

Questionário de apresentação (baseado em Peer Instruction, de Eric Mazur)

1- O quê você espera aprender neste curso?

2- O que você espera fazer com este conhecimento?

3- O que você espera que as aulas façam por você?

4- O que você espera que o livro faça por você?

5- Quantas horas você imagina serão necessárias para aprender tudo que você precisa saber

sobre este curso ? inclua tudo (dever de casa, aulas, etc..)______________horas por semana.

Formato geral da nossa aula:

1) pergunta feita;

2) Estudantes têm tempo para pensar;

3) Estudantes registram ou relatam respostas individuais;

4) Estudantes vizinhos discutem suas respostas;

5) Estudantes registram ou relatam as sua respostas revistas;

6) Feedback para o professor: distribuição de respostas;

7) Explicação da resposta correta;

Dicas para a aula:

1) Leia o tópico a ser apresentado ANTES da aula;

2) não é necessário copiar o material do quadro. Está tudo no livro! Você pode fotocopiar as notas de aula se

desejar;

3) seja ativo!

Page 4: Topicos de Eletromagnetismo I

FIW 591 – Tópicos de Eletromagnetismo I

Nome

Thales de Mileto (grego)

William Gilbert (inglês)

Benjamin Franklin (americano)

Charles Augustin de Coulomb (francês)

Karl Friedrich Gauss (alemão)

Alessandro Volta (italiano)

Hans Christian Oersted (dinamarquês)

André Marie Ampère (francês)

Tópicos de Eletromagnetismo I – Prof. Antônio Carlos

Alguns pioneiros do Eletromagnetismo

Datas contribuição

636-546 a.C.

Percebeu que quando o âmbar é atritado com seda produz pequenas descargas e possuía o poder “mágico” de atrair partículas de palha e penugem. Em grego âmbar = elektron. Também notou o poder atrativo de algumas pedras encontradas em Magnésia, de onde vem o nome magnetismo.

1540-1603 d.C

Realizou os primeiros experimentos de forma sistemática sobre eletricidade e magnetismo descritos no livro Inventou o eletroscópio e foi o primeiro a reconhecer que a Terra era um grande imã, inspirando os princípios da bússola

1706-1790 Cientística e político americano. Seus experimentos o levaram a inventar o pára-ráios. Estabeleceu a lei de conservação da carga e as chamou de positiva e negativa.

1736-1806 Publicou 7 tratados sobre a Eletricidade e o Magnetismo, e outros sobre os fenômenos de torção, o atrito entre sólidos etc. Experimentador genial e rigoroso, realizou uma experiência histórica com uma balança de torção para determinar a força exercida entre duas cargas elétricas (Lei de Coulomb).Durante os últimos quatro anos da sua vida, foi inspetor geral do Ensino Público e teve um papel importante no sistema educativo da época.

1777-1851 Formulou o teorema da divergência relacionasua superfície, a lei de Gauss é a lei que estabelece a relação entre o fluxo elétrico que passa através de uma superfície fechada e a quantidade de carga elétrica que existe dentro do volume limitado por esta superfície. Em 1840, publicou seu influente Dioptrische Untersuchungen, no qual fez a primeira análise sistemática da formação de imagens sob aparaxial.

1745-1827 Por volta de 1800, Volta inventou a célula voltaicavárias em série, inventou a bateria. Em setembro de 1801, Volta viajou até Paris aceitando um convite do próprio imperador Napoleão Bonaparte, para mostra as características de seu invento (a pilha) no Institut de France. E, em honra ao seu trabalho no campo de eletricidade, Napoleão nomeou Vconde em 1810. Em 1815, o imperador da Áustria nomeou Volta professor de filosofia na Universidade de Pádua.

1777-1855 Enquanto se preparava para uma palestra na tarde de 21 de Abril de 1820, Oersted reparou que a agulha de uma bússoladefletia quando uma corrente elétrica era ligada e desligada. Esta deflexão convenceu-o que os campos magnéticos radiam a partir de todos os lados de um fio carregando uma corrente elétrica, tal como ocorre com a luz e o caloconfirmava uma relação direta entre eletricidade e magnetismoInfluenciou o desenvolvimento de uma forma matemática única que representasse as forças magnéticas entre condutores portadores de corrente por parte do físico francês AndréAmpère.

1775-1836 Partindo das experiências feitas pelo dinamarquês Hans Christian Oersted sobre o efeito magnético da corrente elétrica, soube estruturar e criar a teoria que possibilitou a construção de um grande número de aparelhos eletromagnéticos. Além disso descobriu as leis que regem as atrações e repulsões das correntes elétricas entre si. Idealizou o galvanômetro, inventou o primeiro telégrafo elétrico e, em colaboração com Arago, o eletroímã. Inventou também o solenóide.

4

Percebeu que quando o âmbar é atritado com seda produz pequenas descargas e possuía o poder “mágico” de atrair partículas de palha e penugem. Em grego âmbar = elektron.

umas pedras encontradas tismo.

Realizou os primeiros experimentos de forma sistemática sobre eletricidade e magnetismo descritos no livro De Magnete. Inventou o eletroscópio e foi o primeiro a reconhecer que a Terra era um grande imã, inspirando os princípios da bússola

Seus experimentos o levaram a Estabeleceu a lei de conservação da carga

Publicou 7 tratados sobre a Eletricidade e o Magnetismo, e outros sobre os fenômenos de torção, o atrito entre sólidos etc. Experimentador genial e rigoroso, realizou uma experiência histórica com uma balança de torção para determinar a força exercida entre duas cargas elétricas (Lei de Coulomb).Durante os últimos quatro anos da sua vida, foi inspetor geral do Ensino

teve um papel importante no sistema educativo da

Formulou o teorema da divergência relacionando o volume e a lei de Gauss é a lei que estabelece a relação

através de uma superfície fechada e a quantidade de carga elétrica que existe dentro do

. Em 1840, publicou seu , no qual fez a primeira

análise sistemática da formação de imagens sob a aproximação

nventou a célula voltaica e, conectando Em setembro de 1801, Volta

viajou até Paris aceitando um convite do próprio imperador Napoleão Bonaparte, para mostra as características de seu

. E, em honra ao seu trabalho no campo de eletricidade, Napoleão nomeou Volta

Em 1815, o imperador da Áustria nomeou Volta professor de filosofia na Universidade de Pádua.

Enquanto se preparava para uma palestra na tarde de 21 de que a agulha de uma bússola se

era ligada e desligada. o que os campos magnéticos radiam a

fio carregando uma corrente trica, tal como ocorre com a luz e o calor, e que isso

tricidade e magnetismo. o desenvolvimento de uma forma matemática única

que representasse as forças magnéticas entre condutores portadores de corrente por parte do físico francês André-Marie

Partindo das experiências feitas pelo dinamarquês Hans sobre o efeito magnético da corrente elétrica,

soube estruturar e criar a teoria que possibilitou a construção de um grande número de aparelhos eletromagnéticos. Além disso descobriu as leis que regem as atrações e repulsões das

si. Idealizou o galvanômetro, inventou elétrico e, em colaboração com Arago, o

Page 5: Topicos de Eletromagnetismo I

5 FIW 591 – Tópicos de Eletromagnetismo I – Prof. Antônio Carlos

Joseph Henry

1797-1878 Em 1830, enquanto construía eletroimãs, descobriu o fenômeno eletromagnético chamado indução electromagnética ou auto-indutância e a indutância mútua. O seu trabalho foi desenvolvido independentemente de Michael Faraday, mas é a este último que se atribuí a honra da descoberta por ter publicado primeiro as suas conclusões. A Henry também é creditada a invenção do motor elétrico, embora mais uma vez não tenha sido o primeiro a registrar a patente. Seus estudos acerca do relê eletromagético foram a base do telégrafo elétrico, inventado por Morse e Wheatstone. Mais tarde provou que as correntes podem ser induzidas à distância, magnetizando uma agulha com a ajuda de um relâmpago a 13 km de distância.

James P. Joule

1818-1889 (pronuncia-se /ˈdˈuˈl/[Jule]). Estabeleceu que o aquecimento é proporcional ao quadrado da corrente

James Clerck Maxwell (britânico)

1831-1879 Estabeleceu de maneira profunda e elegante a interdependência entre eletricidade e magnetismo. Postulou que a luz era de natureza eletromagnética e que outros comprimentos de onda poderiam existir.

Heinrich Hertz

1857-1894 Pai do rádio, Hertz gerou e detectou ondas de rádio. Hertz demonstrou que, exceto por diferenças no comprimento de onda, a polarização, reflexão e refração de ondas de rádioeram idênticas à luz. Mas sua invenção permaneceu como uma curiosidade de laboratório até que Marconi adicionou um sintonizador e uma grande antena.

Guglielmo Marconi

1874-1937 Inventor do primeiro sistema prático de telegrafia sem fios, em 1896. Marconi se baseou em estudos apresentados em 1897 por Nikola Tesla para em 1899 realizar a primeira transmissão pelo canal da mancha. A teoria de que as ondas electromagnéticas poderiam propagar-se no espaço, formulada por Maxwell, e comprovada pelas experiências de Hertz, em 1888, foi utilizada por Marconi entre 1894 e 1895.

Thomas A. Edison (americano)

1847-1931 Transformou a eletricidade e o magnetismo em aplicações práticas em telegrafia, telefonia, iluminação e geração e transmissão de energia. O Feiticeiro de Menlo Park (The Wizard of Menlo Park), como era conhecido, foi um dos primeiros inventores a aplicar os princípios da produção maciça ao processo da invenção.

Nikola Tesla (Iuguslávo)

1856-1943 Demonstrou o valor das correntes alternadas e inventou o motor de indução. Projetou sistema de potência em Niagara Falls.

Albert Einstein (alemão)

1879-1955 Tornou as equações de Maxwell universais através da teoria da relatividade.

Page 6: Topicos de Eletromagnetismo I

6 FIW 591 – Tópicos de Eletromagnetismo I – Prof. Antônio Carlos

Formulário

Equações de Maxwell Forma diferencial

t

BE

B

Jt

DH

D

∂∂

−=×∇

=⋅∇

=∂∂

−×∇

=⋅∇

rrr

rr

rr

rr

r

0

ρ

Forma integral

0..

0.

...

.

∫ ∫∫

∫∫

∫∫∫ ∫∫

∫∫ ∫∫∫

=+

=

=−

=

C S

S

SC S

S V

SdBdt

dldE

SdB

SdjSdDdt

dldH

dVSdD

rrrr

rr

rrrrrr

rrρ

Equações constitutivas

Ej

HB

ED

rr

rtr

rtr

σ

µ

ε

=

=

=

Campos auxiliares

HM

EP

MB

H

PED

m

eo

o

o

rr

rr

rr

r

rrr

χ

χε

µ

ε

=

=

−=

+=

Força de Lorentz

( )t

pdVBjEF

tBjEf

mec

V

mec

∂∂

=×+=

∂℘∂

=×+=

∫∫∫r

rrrr

rrrrr

ρ

ρ

Lei de Biot-Savart

∫∫

∫∫∫

×=

×=

×=

×=

S

V

C

dSr

rB

dVr

rjB

r

rldIB

r

rvqB

4

4

ˆ'

4

ˆ

4

2

2

2

2

κπµ

πµ

πµπµ

rr

rr

rr

rr

Condições de contorno

tjn

En

Bn

Hn

Dn

∂∂

−=∆

=∆×

=∆

=∆×

=∆

σ

κ

σ

r

rr

r

rr

r

0.ˆ

ˆ

Potenciais

jA

r

rmrA

dVrr

rJA

AB

t

AVE

dVrr

rrV

V

o

Vo

rrr

rr

rr

rrr

rrr

rrr

rr

rr

µπµ

πµ

ρπε

−=⋅∇

×=

−=

×∇=

∂∂

−∇−=

−=

∫∫∫

∫∫∫

2

2

'

ˆ

4)(

'

)'(

4

''

)'(

4

1)(

Energia, momento

( )

2

.

..2

1

2

oo

VV

emem

em

V

EcIS

dVSdVp

SBD

Ejw

udVU

BHDEu

wt

uS

uvS

HES

ε

µε

µε

=≡

=℘=

=×=℘

=

=

+=

−=∂∂

+⋅∇

=

×=

∫∫∫∫∫∫

∫∫∫

rrr

rrrr

rr

rrrr

rr

rrr

Ondas

εµ

ω

µε

=

×=

=∂∂

−∇

Z

BEk

vBE

t

EE

rrr

rrr

rrr

02

Valores numéricos Carga do elétron (módulo):

e =1,6×10-19 C Permeabilidade do vácuo:

µo = 4π×10-7 H/m Permissividade do vácuo:

εo = 8,854×10-12 F/m

1/4πεo = 8,988×109 Nm2/C2 Velocidade da luz no vácuo:

c =(εoµo)-1 = 2,998×108 m/s

Zo =(µo/εo)1/2

= 120πΩ ≅ 377Ω

Page 7: Topicos de Eletromagnetismo I

7 FIW 591 – Tópicos de Eletromagnetismo I – Prof. Antônio Carlos

Formulário

Delta de Dirac

[ ]

( )∫ ∈−=−

−=

−++=−

=

R

ox

n

nn

on

n

Rxdx

fddxxx

dx

dxf

xxdx

dx

axaxa

ax

a

rara

o

)1()()(

)()(

)()(2

1)(

)()(

22

δ

δδ

δδδ

δδ

rr

Relações entre os unitários

( ) ( )( ) ( )

( ) ( )( ) ( )

θθθ

φφθφθφθ

φφθφθφθ

φφφ

φφρ

φφρφ

φφρφ

ˆˆcos

ˆcosˆcosˆˆ

ˆˆcoscosˆcosˆ

ˆ

ˆcosˆˆ

ˆˆcosˆ

ˆ

ˆcosˆˆ

ˆˆcosˆ

senrz

senrsenseny

senrsenx

zz

yxsen

ysenx

zz

seny

senx

−=

++=

−+=

=

+−=

+=

=

+=

−=

)

)

)

Coordenadas cartesianas

zz

fy

y

fx

x

ff

dxdydzdV

zdxdyydxdzxdydzSd

zdzydyxdxld

ˆˆˆ

ˆˆˆ

ˆˆˆ

∂∂

+∂∂

+∂∂

=∇

=

++=

++=

r

r

r

Coordenadas cilíndricas

( ) ( ) ( )

)()()(1

)(

ˆˆ21

ˆ21

ˆˆ1ˆ

ˆˆˆ

ˆˆˆ

2

2

2

2

22

oooo

z

zzrr

zvvv

vv

vvv

zz

ffff

dzdddV

zdddzddzdSd

zdzddld

φφδδρρδρ

δ

φφρ

ρφρ

φφρ

ρρ

φρρφρρφρρφρ

φφρρρ

φρ

φφ

ρρ

−−−=−

∇+

∂−∇+

∂+−∇=∇

∂+

∂+

∂=∇

=

++=

++=

rr

rr

r

r

r

Coordenadas esféricas

( ) ( )( ) ( ) ( )

)()cos(cos)(1

)()()(csc

)(

ˆcscˆ1ˆ

ˆˆˆ

ˆˆˆ

2

2

2

2

ooo

oooo

rrr

rrr

rr

f

r

f

rr

r

ff

ddrdsenrdV

rdrddrdrsenrddsenrSd

drsenrdrdrld

φφδθθδδ

φφδθθδδθ

δ

φφ

θθ

θ

φθθ

φθθφθφθθ

φφθθθ

−−−=

−−−=−

∂∂

+∂∂

+∂∂

=∇

=

++=

++=

rr

r

r

r

vetores

)()()(

)()()(

BACCABCBA

ACBBACCBArrrrrrrrr

rrrrrrrrr

⋅−⋅=××

×⋅=×⋅=×⋅

Operador Nabla

AAA

f

A

ABBABAABBA

fAAfAf

BAABBA

fAAfAf

ABBA

ABBABA

fggffg

rrr

r

r

rrrrrrrrrr

rrr

rrrrrr

rrr

rrrr

rrrrrr

2)()(

0)(

0)(

)()()()()(

)()()(

)()()(

)()()(

)()(

)()()(

)(

∇−⋅∇∇=×∇×∇

=∇×∇

=×∇⋅∇

⋅∇−⋅∇+∇⋅−∇⋅=××∇

∇×−×∇=×∇

×∇⋅−×∇⋅=×∇

∇⋅+⋅∇=∇

∇⋅+∇⋅+

+×∇×+×∇×=⋅∇

∇+∇=∇

Integrais

( )

( )

( )

( )

∫∫ ∫

∫∫ ∫

∫∫∫∫∫

∫∫∫∫∫

∫∫∫∫∫

=∇×

⋅=×∇

=∇

×=×∇

⋅=⋅∇

S C

S C

SV

SV

SV

lfdfSd

ldvSdv

SfddVf

vSddVv

SdvdVv

rrr

rrrrr

rr

rrrr

rrrr

Page 8: Topicos de Eletromagnetismo I

8 FIW 591 – Tópicos de Eletromagnetismo I – Prof. Antônio Carlos

Aula 1 – Álgebra vetorial

Nome:______________________________________________________________________________

Dados os vetores A = 1i – 2k e B = -1 i + 1j . Calcule:

1) A - 2B

A) ( ) +3i - 2j -2k;

B) ( ) -3i +2j -2k;

C) ( ) +3i +2j –2k;

D) ( ) -3i -2j – 2k;

2) A.B

A) ( ) +1;

B) ( ) -1;

C) ( ) 2;

D) ( )-2;

3) A××××B

A) ( ) -2i +2j -1k;

B) ( ) 2i - 2j -1k;

C) ( ) 2i +2j +1k;

D) ( ) 2i +2j -1k;

4) Qual o ângulo entre os vetores A e B no exercício anterior?

A) ( ) cos-1

(10-1/2

);

B) ( ) cos-1

(10+1/2

);

C) ( ) cos-1

(-10-1/2

);

D) ( ) cos-1

(-10+1/2

);

Page 9: Topicos de Eletromagnetismo I

FIW 591 – Tópicos de Eletromagnetismo I

Nome:______________________________________________________________________________

1- O gradiente de f(x,y,z) = x

A) ( ) 2xi+2yj+2z

B) ( ) 2i+2j+2k;

C) ( ) xi+yj+zk;

D) ( ) 2xi-2yj+2z

2- Qual a opção que melhor descreve o gradiente no ponto A da função representada pelas curvas

de nível abaixo?

A) ( ) ↑;

B) ( ) ↓;

C) ( ) →;

D) ( ) ←;

3- A divergência de v = x2 i

A) ( ) 2x-2z;

B) ( ) 2x+2z;

C) ( ) 2x-3z;

D) ( ) 2x-2x;

4- Qual das opções abaixo representa uma função com divergência positiva?

A)( )

5- Dado um campo magnético

representa as linhas de campo d

A) ( ); B) ( );

6- O rotacionai de xyi+yzj+zx

A) ( ) +yi+ zj +xk;

B) ( ) –yi+ zj +xk;

C) ( ) –yi-zj-xk;

D) ( ) +yi +zj -xk;

Tópicos de Eletromagnetismo I – Prof. Antônio Carlos

Aula 2 – Cálculo diferencial I

Nome:______________________________________________________________________________

f(x,y,z) = x2 + y

2 + z

2 é:

+2zk;

+2zk;

Qual a opção que melhor descreve o gradiente no ponto A da função representada pelas curvas

i + 3xz2 j -2xz k é:

Qual das opções abaixo representa uma função com divergência positiva?

B)( )

C)( )

D)( )

Dado um campo magnético B=Bok=rot A, qual das opções da figura do item anterior

representa as linhas de campo de A?

C) ( ); D) ( );

+zxk é:

9

Nome:______________________________________________________________________________

Qual a opção que melhor descreve o gradiente no ponto A da função representada pelas curvas

D)( )

, qual das opções da figura do item anterior melhor

Page 10: Topicos de Eletromagnetismo I

FIW 591 – Tópicos de Eletromagnetismo I

Nome:______________________________________________________________________________

1- O Laplaciano da função

A) ( ) –senx.seny.senz;

B) ( ) +senx.seny.senz;

C) ( ) +3senx.seny.senz;

D) ( ) –3senx.seny.senz;

2- Dos campos vetoriais abaixo, quais podem ser gradiente de uma função?

I)

A) ( ) todos;

B) ( ) nenhum;

C) ( )somente I;

D) ( )somente II;

E) ( ) somente III;

F) ( ) somente I e II;

3- Seja B=rotA , então podemos afirmar que necessariamente:

A) ( ) divB =0;

B) ( ) divA =0;

C) ( ) rotB =0;

D) ( ) grad(divB) =0;

Tópicos de Eletromagnetismo I – Prof. Antônio Carlos

Aula 3 – cálculo diferencial II

Nome:______________________________________________________________________________

O Laplaciano da função g(x, y, z)=senx.seny.senz é:

senx.seny.senz;

( ) +senx.seny.senz;

( ) +3senx.seny.senz;

3senx.seny.senz;

abaixo, quais podem ser gradiente de uma função?

II) III)

, então podemos afirmar que necessariamente:

10

Nome:______________________________________________________________________________

Page 11: Topicos de Eletromagnetismo I

11 FIW 591 – Tópicos de Eletromagnetismo I – Prof. Antônio Carlos

Aula 3 – cálculo diferencial II (para casa)

Nome:______________________________________________________________________________

1- Prove que a divergência de um rotacional é sempre zero. Verifique para a função va = x2 i + 3xz

2

j – 2xz k.

2- Prove que o rotacional de um gradiente é sempre zero. Verifique para a função f=x2y

3z

4

Page 12: Topicos de Eletromagnetismo I

12 FIW 591 – Tópicos de Eletromagnetismo I – Prof. Antônio Carlos

Aula 4 – Cálculo Integral

Nome:______________________________________________________________________________

1 – A integral de linha da função v = x2 i + 2yz j +y

2 k da origem (0,0,0) até o ponto (1,0,0) ao longo do

eixo x resulta em:

A) ( ) 1;

B) ( ) -1;

C) ( ) 1/3;

D) ( ) -1/3;

2-A integral de linha da função v = x2 i + 2yz j +y

2 k da origem (0,0,0) até o ponto (0,1,0) ao longo do eixo

y resulta em: A) ( ) 1;

B) ( ) -1;

C) ( ) 2;

D) ( ) 0;

3- O vetor dS que da superfície quadrada de arestas (0,0,0), (0,1,0), (0,1,1) e (0,0,1) é igual a: A) ( ) dxdyk; B) ( ) dydzi; C) ( ) dxdzj; D) ( ) dxdzk;

4- O fluxo da função vetorial D=x

2yi+y

2xj+zk através da superfície quadrada de arestas (0,0,0),

(0,1,0), (0,1,1) e (0,0,1) é igual a: A) ( ) 0; B) ( ) 1; C) ( )-1; D) ( ) 2;

5- A integral de volume da função constante ρ para a centrada na origem e raio R é. A) ( ) 4ρπR

3;

B) ( ) ρπR3;

C) ( ) (4/3)ρπR3;

D) ( ) 3ρπR3;

6- (Teorema de Gauss) Se div E=ρ/ε, então:

A) ( ) ( )∫∫ ∫∫∫=×∇S V

dVdSE ρε .rr

;

B) ( ) ∫∫ ∫∫∫=S V

dVSdE ρεrr

. ;

C) ( ) ∫ ∫∫∫=V

dVldE ρεrr

. ;

D) ( ) [ ]∫∫ ∫∫∫ ∇=S V

dVSdE ρεrrr

. ;

Page 13: Topicos de Eletromagnetismo I

13 FIW 591 – Tópicos de Eletromagnetismo I – Prof. Antônio Carlos

7- (Teorema de Stokes) Se rotB=µj, então:

A) ( ) ∫ ∫=C C

ldjldBrrrr

.. µ ;

B) ( ) ∫ ∫∫=C S

SdjldBrrrr

.. µ ;

C) ( ) ( )∫ ∫∫=×∇C S

SdjldBrrrrr

.. µ ;

D) ( ) ( )∫ ∫∫ ×∇=C S

SdjldBrrrrr

.. µ ;

8- Se rotC=0 então:

A) ( ) divC=0;

B) ( ) C=gradV;

C) ( ) a integral de caminho de C ao longo de uma curva fechado é positiva;

D) ( ) a integral de caminho de C ao longo de uma curva fechado é negativa;

Para casa: Probs. 1.53, 1.54, 1.55, 1.56, 1.57, 1.58

Page 14: Topicos de Eletromagnetismo I

FIW 591 – Tópicos de Eletromagnetismo I

Adaptado de McDermottt, Shaffer,

1- Pegue uma folha de papel.

Qual linha ou segmento de reta que você usaria para especificar a

modo que qualquer pessoa possa manter o papel no mesmo plano ou em um plano paralelo ao

seu?

2- A área de uma superfície plana pode ser representada por um único

A. O que a direção deste vetor representa?

3- O que você esperaria que a magnitude deste vetor representasse?

4- Coloque uma folha de papel

quadriculado sobre uma mesa.

Descreva a direção e a magnitude

do vetor área para a folha de

papel.

5- Dobre a folha duas vezes de modo

a formar um tubo triangular oco.

folha inteira pode ser representada

por um único vetor área? Se não

qual é o número mínimo de vetores

necessários?

6- Dobre agora a folha de modo a

formar um tubo oco cilíndrico.

orientação de cada quadrado que

compõe a folha pode ser

representada por

Explique

Tópicos de Eletromagnetismo I – Prof. Antônio Carlos

Aula 4 – para casa

McDermottt, Shaffer, & P.E. G. U. Wash,Tutorials in Introductory Physics

Área como um vetor

Pegue uma folha de papel. A folha pode ser vista com parte de uma superfície plana maior.

Qual linha ou segmento de reta que você usaria para especificar a orientação da folha, de

modo que qualquer pessoa possa manter o papel no mesmo plano ou em um plano paralelo ao

A área de uma superfície plana pode ser representada por um único vetor, chamado vetor área

. O que a direção deste vetor representa?

e você esperaria que a magnitude deste vetor representasse?

Coloque uma folha de papel

quadriculado sobre uma mesa.

Descreva a direção e a magnitude

do vetor área para a folha de

Dobre a folha duas vezes de modo

a formar um tubo triangular oco. A

folha inteira pode ser representada

por um único vetor área? Se não

qual é o número mínimo de vetores

Dobre agora a folha de modo a

formar um tubo oco cilíndrico. A

orientação de cada quadrado que

compõe a folha pode ser

representada por um vetor dA?

14

Tutorials in Introductory Physics

A folha pode ser vista com parte de uma superfície plana maior.

orientação da folha, de

modo que qualquer pessoa possa manter o papel no mesmo plano ou em um plano paralelo ao

, chamado vetor área

Page 15: Topicos de Eletromagnetismo I

15 FIW 591 – Tópicos de Eletromagnetismo I – Prof. Antônio Carlos

Aula 5 – a função delta de Dirac, sistema de coordenadas

Nome:______________________________________________________________________________

1- A integral ∫ −−−6

2

2 )3()123( dxxxx δ resulta em :

A) ( ) 19;

B) ( ) 20;

C) ( ) 21;

D) ( ) 22;

E) ( ) 0;

2- A integral ∫ +3

0

3 )1( dxxx δ

resulta em:

A) ( ) 1;

B) ( ) -1;

C) ( ) 0;

F) ( ) 2;

3- Qual é a dimensão da função δ(x) se x é dado em metros:

A) ( ) [δ(x)] = m;

B) ( ) [δ(x)] = adimensional;

C) ( ) [δ(x)] = m-1

;

4- Qual é a dimensão da função δ(x)δ(y)δ(z)=δ(r)= δ3(r)= se x,y e z são expressos em metros:

A) ( ) [δ(x)] = m3 ;

B) ( ) [δ(x)] = adimensional;

C) ( ) [δ(x)] = m-3

;

5- Uma carga pontual q se encontra na posição (3,2,-1). A densidade volumar de carga ρ=dq/dV é

dada por:

A) ( ) ρ=q;

B) ( ) ρ=∞;

C) ( ) ρ =qδ(x-3)δ(y-2)δ(z-1);

D) ( ) ρ =qδ(x+3)δ(y+2)δ(z-1);

E) ( ) ρ =qδ(x-3)δ(y-2)δ(z+1);

6- O vetor deslocamento r é representado em coordenadas cilíndricas. Nesta representação,

encontre o vetor velocidade

A – ( )

B- ( )

C- ( )

D - ( )

Para casa: probls. 1.36, 1.37, 1.38, 1.39, 1.40, 1.41, 1.42, 1.44, 1.45, 1.46, 1.47,1.48, 1.53, 1.54, 1.55,

1.56, 1.57, 1.58

Page 16: Topicos de Eletromagnetismo I

16 FIW 591 – Tópicos de Eletromagnetismo I – Prof. Antônio Carlos

Aula 6 – O campo elétrico

Nome:______________________________________________________________________________

1- Duas esferas de chumbo idênticas, pequenas, são separadas pela distância de 1 m. As esferas tinham originalmente a mesma carga positiva e a força entre elas é Fo. Metade da carga de uma esfera é então deslocada para a outra esfera. A força entre as esferas será A- ( ) Fo/4 ; B- ( ) Fo/2 ; C- ( ) 3Fo/4 ; D- ( ) 3Fo/2 ; E- ( ) 3Fo ;

2- Doze cargas iguais, q, estão situadas nos vértices de polígono regular de 12 lados ( por

exemplo, uma em cada número de um relógio de ponteiros) definido por um círculo de raio R.

Qual a força total sobre uma carga de prova q no centro [k=(4πεo)-1

] ?

A) ( ) zero;

B) ( ) kq2/R

2;

C) ( ) 12kq2/R

2;

D) ( ) 2kq2/R

2;

E) ( ) 6kq2/R

2;

3- Suponha que uma das 12 cargas é removida (a que estava na posição 6 horas). Qual é a força

sobre q?

A) ( ) zero;

B) ( ) kq2/R

2;

C) ( ) 12kq2/R

2;

D) ( ) 2kq2/R

2;

E) ( ) 6kq2/R

2;

4- Agora 13 cargas iguais, q, são dispostas de polígono regular de 13 lados. Qual é a força sobre a

carga de prova q no centro?

A) ( ) zero;

B) ( ) kq2/R

2;

C) ( ) 12kq2/R

2;

D) ( ) 2kq2/R

2;

E) ( ) 6kq2/R

2;

5- Se uma das 13 cargas é removida, qual é a força sobre q? Explique o seu raciocínio.

A) ( ) zero;

B) ( ) kq2/R

2;

C) ( ) 12kq2/R

2;

D) ( ) 2kq2/R

2;

E) ( ) 6kq2/R

2;

6- Quatro partículas puntiformes, de mesma carga q, situam-se nos vértices de um quadrado do

plano xy com centro na origem e com lados, de comprimento 2a, paralelos aos eixos desse

plano. Determine (a) a expressão cartesiana da densidade volumar de cargas; (b) a carga total,

integrando a densidade de cargas. Resp: (a) ρ(r)=q[δ(x+a)+ δ(x-a)] [δ(y+a)+ δ(y-a)] [δ(z)]; (b) qtot = 4q

A) ( ) ρ(r)=q[δ(x+a)- δ(x-a)] [δ(y+a)+ δ(y-a)] [δ(z)]; qtot = 4q

B) ( ) ρ(r)=q[δ(x+a)+ δ(x-a)] [δ(y+a)- δ(y-a)] [δ(z)]; qtot = 4q

C) ( ) ρ(r)=q[δ(x+a)+ δ(x-a)] [δ(y+a)+ δ(y+a)] [δ(z)]; qtot = 4q

D) ( ) ρ(r)=q[δ(x+a)+ δ(x-a)] [δ(y+a)+ δ(y-a)] [δ(z)]; qtot = 4q

Page 17: Topicos de Eletromagnetismo I

17 FIW 591 – Tópicos de Eletromagnetismo I – Prof. Antônio Carlos

Aula 6 – O campo elétrico

Para casa: Probs. 2.2, 2.3, 2.4, 2.5, 2.6, 2.7, 2.8

Curiosidades sobre o campo elétrico: Fogo-de-santelmo. Santo Elmo é o padroeiro dos

marinheiros. Conta-se que os marinheiros do passado atribuíam a um fenômeno eletrostático

um significado divino- a aparição do referido santo. Na realidade, o que a crença dos antigos

acabou endeusando é o fenômeno conhecido por efeito corona. Os mastros dos navios eram

envoltos por uma luminosidade suave, resultado da emissão de luz na recombinação de íons e

elétrons. As nuvens eletrizadas provocavam a indução de cargas elétricas nas pontas dos

mastros. O intenso campo elétrico nas vizinhanças das pontas ionizava as partículas de ar que,

posteriormente, emitiam a luz durante a recombinação. A superstição acabou denominando o

fenômeno fogo-de-santelmo. O mesmo efeito corona pode também ser observado, por

exemplo, em linhas de transmissão elétrica com sobrecarga, que ficam envoltas por uma

luminosidade ao longo de sua extensão. Retirado de Carlos, Kazuhito, Fuke, Os alicerces da

Física vol.3

Page 18: Topicos de Eletromagnetismo I

18 FIW 591 – Tópicos de Eletromagnetismo I – Prof. Antônio Carlos

Aula 7 – A divergência e o rotacional dos campos eletrostáticos

Nome:______________________________________________________________________________

1- Uma carga q está uniformemente distribuída no volume de uma esfera de raio R com centro

em ro. Qual o fluxo do campo elétrico sobre uma superfície de raio r<R de mesmo centro?

a) ( ) q/εo; b) ( ) (q/εo)(r/R)3; c) ( ) (q/εo)(R/r)

3; d) ( ) 0;

2- Uma carga q está uniformemente distribuída no volume de uma esfera oca de raio interno a e

raio externo R com centro em ro. Qual o fluxo do campo elétrico sobre uma superfície de raio

r<a de mesmo centro?

a) ( ) q/εo; b) ( ) (q/εo)(r/R)3; c) ( ) (q/εo)(R/r)

3; d) ( ) 0;

3- Qual o fluxo do campo elétrico da esfera da questão 2 sobre uma superfície de raio a<r<R?

a) ( ) q/εo; b) ( )(q/εo)(r3-a

3)/(R

3-a

3) c) ( ) (q/εo)(r/R)

3; d) ( ) 0;

4- Ainda sobre a esfera da questão 2, qual o fluxo do campo elétrico sobre uma superfície de raio

r>R?

a) ( ) q/εo; b) ( )(q/εo)(r3-a

3)/(R

3-a

3) c) ( ) (q/εo)(r/R)

3; d) ( ) 0;

5- Prova de acesso ao Mestrado Profissional em Ensino de Física (UFRJ/2011):

6- Seleção ao mestrado em Ensino (2013)

Page 19: Topicos de Eletromagnetismo I

FIW 591 – Tópicos de Eletromagnetismo I

7- Três lâminas infinitas

uniformemente carregadas são

colocadas lado a lado conforme

mostrado na figura ao lado.

Considere as densidades

superficiais de cargas das lâminas

indicadas na figura , com

Qual das opções abaixo

corresponde ao campo elétrico

resultante nas posições 1,2,3 e 4

indicadas na figura?

A) ( ) E1 =-3σ/εoi, E2

= σ/εoi, E4 =3σ/εoi;

B) ( ) E1 =3σ/2εoi, E2

= -σ/2εoi, E4 =-3σ/2

C) ( ) E1 =σ/2εoi, E2 =3

= -3σ/2εoi, E4 =-σ/2

D) ( ) E1 =-σ/2εoi, E2

E3 = -σ/2εoi, E4 =σ/2

E) ( ) E1 =σ/εoi, E2 =-3

-3σ/εoi, E4 =σ/εoi;

F) Nenhuma das respostas

anteriores.

8- Dois planos não condutores

extensão infinita e

perpendiculares entre si estão

uniformemente carregados com

uma densidade superficial de

carga σ>0. Assinale em qual dos

quadrantes as linhas de força

associadas ao campo elétrico

estão representadas

corretamente.

A) ( ) I;

B) ( ) II;

C) ( ) III;

D) ( ) IV;

Para casa: probls. 2.9, 2.10, 2.11, 2.12, 2.13, 2.14, 2.15, 2.16, 2.17, 2.18, 2.50

Atenção! Este tópico apresenta alto índice de erro

nas avaliações.

Tópicos de Eletromagnetismo I – Prof. Antônio Carlos

Três lâminas infinitas

uniformemente carregadas são

colocadas lado a lado conforme

mostrado na figura ao lado.

Considere as densidades

superficiais de cargas das lâminas

indicadas na figura , com σ>0.

Qual das opções abaixo

corresponde ao campo elétrico

resultante nas posições 1,2,3 e 4

2 =-σ/εoi, E3

=σ/2εoi, E3

/2εoi;

=3σ/2εoi, E3

/2εoi;

2 =-3σ/2εoi,

/2εoi;

3σ/εoi, E3 =

Nenhuma das respostas

Dois planos não condutores de

extensão infinita e

perpendiculares entre si estão

uniformemente carregados com

uma densidade superficial de

>0. Assinale em qual dos

quadrantes as linhas de força

associadas ao campo elétrico

estão representadas

Para casa: probls. 2.9, 2.10, 2.11, 2.12, 2.13, 2.14, 2.15, 2.16, 2.17, 2.18, 2.50

Atenção! Este tópico apresenta alto índice de erro

19

Page 20: Topicos de Eletromagnetismo I

20 FIW 591 – Tópicos de Eletromagnetismo I – Prof. Antônio Carlos

Aula 8– Potencial Elétrico

Nome:______________________________________________________________________________

1- O que define um campo conservativo?

A) ( ) ∫ = 0. SdFrr

ou 0. =∇ Frr

B) ( ) A força deve ser de fricção

C) ( ) A força deve ser nuclear

D) ( ) A força deve ser eletromagnética

E) ( ) ∫ = 0. ldFrr

ou 0rrr

=×∇ F

2- Pode-se dizer que o potencial da Terra é de + 100 V em vez de zero? Que efeito teria esta

suposição nos valores medidos de (a) potenciais (b) diferença de potenciais?

A) ( ) sim, não seria alterado, seria alterado;

B) ( ) não, não seria alterado, seria alterado;

C) ( ) não, seria alterado, não seria alterado;

D) ( ) não, seria alterado, seria alterado;

E) ( ) sim, seria alterado, não seria alterado;

3- Um dos campos eletrostáticos abaixo é impossível. Qual? (k é uma constante com dimensões

apropriadas)

A) ( ) E =k[xyi+2yzj+3xzk]

B) ( ) E =k[y2i+(2xy+z

2)j+2yzk]

4- Para o campo possível, calcule o potencial no ponto (x,y,z) utilizando a origem com ponto de

referência. Verifique a sua resposta calculando ∇V. (Dica: você deve escolher um caminho

específico para integrar. Não importa qual o caminho, uma vez que a resposta é independente

do caminho, mas você não pode integrar a menos que tenha um caminho em particular em

mente).

A) ( ) V (x,y,z) = -k( y2x-yz

2)

B) ( ) V (x,y,z) = -k( -y2x+yz

2)

C) ( ) V (x,y,z) = +k( y2x+yz

2)

D) ( ) V (x,y,z) = -k( y2x+yz

2)

Page 21: Topicos de Eletromagnetismo I

FIW 591 – Tópicos de Eletromagnetismo I

Problema 2.28 – Use a equação

de uma esfera sólida de raio R com densidade de carga uniforme e carga total q.

Solução: Um problema importante é o da infinitude do potencial. A solução deste problema

envolve sabermos o que acontece quando o observado

situado dentro da distribuição de cargas, ou seja, quando

o que acontece com a integral acima, vamos escolher como ponto de observação a origem do

sistema de coordenadas. Fisicam

matematicamente r = 0

|r- r’|= r’. Então:

∫∫∫ −=

''

)'(

4

1)(

Vo

dVrr

rrV

ρπε

rr

rr

denominador o pólo r’, cancelado pelo fator r’

esféricas.

Como r e r’ são positivos, sua soma é sempre positiva. A diferença r

=≥

<

>=

−+R

o

u

u

R

qRrV

rr

rr

urr

du

0

3

)(

)0(22

4

3

2

1)(

(2

('2

'

πε

π

Se o observador estiver dentro

nas duas contribuições (r>r’) e (r<r’).

34

3

2

1)(

R

qRrV

o

=≥πε

Tópicos de Eletromagnetismo I – Prof. Antônio Carlos

Aula 8 – Exemplo

Use a equação ''

)'(

4

1)(

'

dVrr

rrV

Vo

∫∫∫ −= rr

rr ρ

πεpara calcular o potencial dentro

de uma esfera sólida de raio R com densidade de carga uniforme e carga total q.

Solução: Um problema importante é o da infinitude do potencial. A solução deste problema

envolve sabermos o que acontece quando o observador resolve medir o potencial num ponto

situado dentro da distribuição de cargas, ou seja, quando r’ →r, ou seja, rrrr= r

o que acontece com a integral acima, vamos escolher como ponto de observação a origem do

sistema de coordenadas. Fisicamente, esse é um ponto como qualquer outro do espaço, mas,

0 simplifica a expressão da distância entre o observador e a fonte para

∫∫∫

='

2

3 '

'''''

4

3

4

1'

Vo r

ddrdsenr

R

qdV

φθθππε

. Como se vê, desaparece do

r’, cancelado pelo fator r’2 do elemento de volume em coordenadas

[ '2''cos'22'

'''

2

2

4

3

4

1)(

'''2

'cos'2

'cos'2

''''2

4

3

4

1)(

''cos'2

'''''

4

3

4

1)(

''cos'2'

0

22

)(

)0(22

0

)(

)0(223

0 02

2

3

'22

2

3

22

rrrrrrurr

du

urr

dudrr

rR

qrV

dsenrrdu

rru

rrrr

dsendrr

R

qrV

rrrr

ddrdsenr

R

qrV

rrrrrr

u

u

R u

uo

R

o

Vo

+=+−=−+

−+

=

−=

=+−

=

+−

=

+−=−

∫ ∫

∫ ∫

∫∫∫

ππ

π

π

θ

πππε

θθθ

θ

θθπ

ππε

θ

φθθππε

θ

r

r

r

rr

Como r e r’ são positivos, sua soma é sempre positiva. A diferença r-r’ pode ter qualquer sinal

=

=

<

>

R

oo r

q

r

R

R

qdr

rr

r

r

0

3

3

2

48

3'

2'

)'

)'

πεπε

bservador estiver dentro volume, a integral do potencial deverá se desmembrar

nas duas contribuições (r>r’) e (r<r’).

( )22

3

0 0

22

33

4

3

6

1'

'

2''

2' rR

R

qdr

rrdr

rr

o

R R

=

+

∫ ∫ πε

21

para calcular o potencial dentro

de uma esfera sólida de raio R com densidade de carga uniforme e carga total q.

Solução: Um problema importante é o da infinitude do potencial. A solução deste problema

r resolve medir o potencial num ponto

r- r’→0.Para vermos

o que acontece com a integral acima, vamos escolher como ponto de observação a origem do

ente, esse é um ponto como qualquer outro do espaço, mas,

simplifica a expressão da distância entre o observador e a fonte para rrrr=

. Como se vê, desaparece do

do elemento de volume em coordenadas

]'

'2

rr

r

−−

r’ pode ter qualquer sinal

volume, a integral do potencial deverá se desmembrar

Page 22: Topicos de Eletromagnetismo I

22 FIW 591 – Tópicos de Eletromagnetismo I – Prof. Antônio Carlos

Aula 8– Para Casa

1-Exame de acesso ao Mestrado Profissional em Ensino de Física (UFRJ/2012):

2-Exame de acesso ao Mestrado Profissional em Ensino de Física (UFRJ/2011):

Page 23: Topicos de Eletromagnetismo I

23 FIW 591 – Tópicos de Eletromagnetismo I – Prof. Antônio Carlos

Aula 8– Para Casa (continuação)

5- Exame de acesso ao Mestrado Profissional em Ensino de Física (UFRJ/2010):

6- Exame de acesso ao Mestrado Profissional em Ensino de Física (UFRJ/2013)

Page 24: Topicos de Eletromagnetismo I

24 FIW 591 – Tópicos de Eletromagnetismo I – Prof. Antônio Carlos

Aula 9 – Trabalho e energia em eletrostática

Nome:______________________________________________________________________________

1- Uma carga q é colocada a uma distância r da origem. Em uma segunda configuração, a carga q

é removida e uma carga 2q é colocada a uma distância 2r. Em ambos os casos há uma carga Q

na origem. Se todas as cargas são positivas, qual carga está em potencial maior?

a) ( ) q b) ( ) 2q c) ( ) as duas cargas possuem o mesmo potencial

2- Qual carga na questão anterior tem uma energia potencial eletrostática maior?

a) ( )Q b) ( )2q c) ( ) as duas cargas possuem a mesma energia potencial

3- No modelo de quark de partículas fundamentais, um próton é composto de três quarks: dois

“up” , cada um com carga 2e/3 e um quark “down”, com carga de –e/3. Suponha que os três

quarks estão eqüidistantes uns dos outros. Assuma essa distância como d e calcule (a) a energia

potencial das interações entre os dois quarks “up” e (b) a energia potencial elétrica total do

sistema [k=(4πεo)-1

] .

A) ( ) 4ke2/9d; +8ke

2/9d;

B) ( ) 4ke2/9d; +4ke

2/9d;

C) ( ) 4ke2/9d; -ke

2/9d;

D) ( ) 4ke2/9d; 0;

E) ( ) 4ke2/9d; -4ke

2/9d;

4- Quais das grandezas eletrostáticas abaixo obedecem ao princípio da superposição

a) ( ) campo elétrico, potencial elétrico e energia;

b) ( ) campo elétrico e potencial elétrico;

c) ( ) potencial elétrico e energia;

d) ( ) campo elétrico e energia;

e) ( ) campo elétrico, potencial elétrico e trabalho;

5- Três cargas (2 positivas e 1 negativa) estão situadas nos cantos de um quadrado (lado a).

Quanto trabalho é necessário para trazer outra carga +q, do infinito até o quarto canto?

Quanto trabalho é necessário para juntar as quatro cargas?

A) ( ) (q2/4πaεo)(2+2

1/2) ; (2q

2/4πaεo)(2+2

1/2) ;

B) ( ) (q2/4πaεo)(-2-2

1/2) ; (2q

2/4πaεo)(-2-2

1/2) ;

C) ( ) (q2/4πaεo)(+2+2

1/2) ; (2q

2/4πaεo)(+2-2

1/2) ;

D) ( ) (q2/4πaεo)(-2-2

1/2) ; (2q

2/4πaεo)(-2+2

1/2) ;

E) ( ) (q2/4πaεo)(-2+2

1/2) ; (2q

2/4πaεo)(-2+2

1/2) ;

para casa: probls. 2.34

Page 25: Topicos de Eletromagnetismo I

25 FIW 591 – Tópicos de Eletromagnetismo I – Prof. Antônio Carlos

Questionário para ser respondido e entregue (não é preciso se identificar) (baseado em Peer Instruction, de

Eric Mazur)

1- O que você gosta nesta aula ?

2- O que você detesta nesta aula?

3- Se você estivesse lecionando este curso, o quê você faria?

4- Se você pudesse mudar algo nesta aula, o quê seria?

Page 26: Topicos de Eletromagnetismo I

FIW 591 – Tópicos de Eletromagnetismo I

Nome:______________________________________________________________________________

1- Duas cavidades esféricas de raios a e b são escavadas no interior de uma esfera condutora neutra de raio

R (figura). No centro de cada cavidade é colocada uma carga pontual

a) As densidades superficiais de carga

A) ( ) –qa/4π

B) ( ) +qa/4π

C) ( ) +qa/4π

D) ( ) –qa/4πb) Qual é o campo fora do condutor

A) ( ) -(qa +qb

B) ( ) (qa -qb)/4

C) ( ) (qa+ qb

D) ( ) 0;

c) Qual é o campo dentro de cada cavidade?

A) ( ) ambos nulos;

B) ( ) –qa/4π

C) ( ) qa/4πε

D) ( ) +qa/4πd) Qual é a força entre q

A) ( ) nula;

B) ( ) –qaqb/4πεo

C) ( ) +qaqb/4πεo

D) ( ) qaqb/8πεo (a+b)

e) Qual dessas respostas mudaria se uma terceira carga, q

do condutor?

A) ( ) as letras a e b;

B) ( ) todas;

C) ( ) nenhuma;

D) ( ) a letra a e d;

E) ( ) somente a letra c;

2- Uma casca esférica condutora e isolada possui carga negativa. O que irá acontecer se um objeto de metal

positivamente carregado é posto em contacto com

carga positiva é (a) menor que, (b) igual a e (c) maior que a carga negativa.

3- Uma camada de metal esférica possui uma distribuição superficial uniforme de cargas. O potencial é o

mesmo sobre a superfície da camada. Qual afirmação é correta?

a) ( ) O potencial é maior no centro geométrico do volume esférico;

b) ( ) O potencial é menor no centro geométrico do volume esférico;

c) ( ) O potencial no centro do volume é o mesmo que o da superfície;

Tópicos de Eletromagnetismo I – Prof. Antônio Carlos

Aula 10 – Condutores

Nome:______________________________________________________________________________

Duas cavidades esféricas de raios a e b são escavadas no interior de uma esfera condutora neutra de raio

R (figura). No centro de cada cavidade é colocada uma carga pontual- chame essas cargas de q

s densidades superficiais de carga σa , σb e σR, são respectivamente.

πa2; +qb/4πa

2; (qa - qb)/4πR

2;

πa2; -qb/4πa

2; -(qa +qb)/4πR

2;

πa2; +qb/4πa

2; (-qa +qb)/4πR

2;

πa2; –qb/4πa

2; (qa +qb)/4πR

2;

Qual é o campo fora do condutor (a uma distância r do centro)?

b)/4πεor2;

)/4πεor2;

b)/4πεor2;

Qual é o campo dentro de cada cavidade?

( ) ambos nulos;

πεoa2; -qb/4πεob

2;

oa2; -qb/4πεob

2;

πεoa2; +qb/4πεob

2;

Qual é a força entre qa e qb ?

(a+b)2;

(a+b)2;

(a+b)2;

Qual dessas respostas mudaria se uma terceira carga, qc, fosse aproximada

( ) as letras a e b;

nenhuma;

( ) a letra a e d;

( ) somente a letra c;

Uma casca esférica condutora e isolada possui carga negativa. O que irá acontecer se um objeto de metal

positivamente carregado é posto em contacto com o interior da casca? Discuta os três casos

carga positiva é (a) menor que, (b) igual a e (c) maior que a carga negativa.

Uma camada de metal esférica possui uma distribuição superficial uniforme de cargas. O potencial é o

mesmo sobre a superfície da camada. Qual afirmação é correta?

O potencial é maior no centro geométrico do volume esférico;

O potencial é menor no centro geométrico do volume esférico;

O potencial no centro do volume é o mesmo que o da superfície;

26

Nome:______________________________________________________________________________

Duas cavidades esféricas de raios a e b são escavadas no interior de uma esfera condutora neutra de raio

chame essas cargas de qa e qb.

, fosse aproximada

Uma casca esférica condutora e isolada possui carga negativa. O que irá acontecer se um objeto de metal

o interior da casca? Discuta os três casos em que a

Uma camada de metal esférica possui uma distribuição superficial uniforme de cargas. O potencial é o

Page 27: Topicos de Eletromagnetismo I

27 FIW 591 – Tópicos de Eletromagnetismo I – Prof. Antônio Carlos

4- A figura mostra o corte transversal de uma cavidade no interior de um condutor elétrico metálico neutro.

Uma carga positiva q está dentro da cavidade. A linha tracejada representa um corte de uma superfície

gaussiana fechada. A superfície gaussiana está no interior do condutor e envolve a cavidade interna.

Marque a afirmativa correta. (a) A carga elétrica no interior da superfície gaussiana é q . (b) O campo elétrico no interior da cavidade é nulo. (c) O campo elétrico no exterior do condutor é nulo. (d) Se o condutor for aterrado, o campo elétrico se anula em

seu exterior.

5- Uma esfera de metal de raio R, contendo uma carga q, é envolvida por uma camada de metal espessa e

concêntrica (raio interno a, raio externo b). A camada não contém nenhuma carga elétrica.

I) Encontre a densidade superficial de carga σ em r=R, em r=a e em r = b.

A) ( ) -q/4πR2; q/4πa

2; q/4πb

2;

B) ( ) q/4πR2; q/4πa

2; q/4πb

2;

C) ( ) q/4πR2; -q/4πa

2; -q/4πb

2;

D) ( ) q/4πR2; -q/4πa

2; q/4πb

2;

II) Encontre o potencial no centro, usando o infinito como ponto de referência.

A) ( ) (-q/4πεo)(1/b+1/a+1/R);

B) ( ) (q/4πεo)(1/b-1/a-1/R);

C) ( ) (q/4πεo)(1/b+1/a+1/R);

D) ( ) (q/4πεo)(1/b+1/a-1/R);

E) ( ) 0;

III) Agora a superfície externa é tocada com um fio aterrado, que abaixo o seu potencial à zero

(mesmo que no infinito). Como as suas respostas em A) e B) mudam?

Atenção! Este tópico apresenta alto índice de erros nas avaliações!

Page 28: Topicos de Eletromagnetismo I

28 FIW 591 – Tópicos de Eletromagnetismo I – Prof. Antônio Carlos

Aula 10 – Condutores (para casa)

1-prova de acesso à pós-graduação em física (UNIPÓS -2012)

Page 29: Topicos de Eletromagnetismo I

FIW 591 – Tópicos de Eletromagnetismo I

Nome:______________________________________________________________________________

1- Uma particula de carga q <uma região de campo magnético uniforme B = BA) +qvBi B) -qvBi C) +qvBj D) -qvBz E) +qvBz

2- Um próton se move na direção +z após ser acelerado a partir doV. O próton então passa através de uma região onde há um campo elétrico uniforme E na direção +x e um campo magnetico uniforme na direção +y, mas a trajetória do próton não é afetada pelos campos. Se o experimento fosse repetido usando uma diferença de potencial de 2V, o próton seria:A) Defletido na direção +x;B) Defletido na direção C) Defletido na direção +y;D) Defletido na direção E) Não seria defletido;

3- Uma corrente de 1 A passa por um fio de 2 mm de diâmetro . O corrente j é: A) 1 A/mm

2;

B) 4/π A/mm2;

C) π-1 A/mm

2;

D) 2/π A/mm2;

E) 0,5 A/mm2;

4- Uma corrente de 0,5 A é transportada na superfície de um lâmina fina e comprida de 5 mm de largura,

conforme ilustrado pela figura abaixo. A densidade liA) 0,1 i A/mm; B) 0,5 i A/mm; C) 0,1 j A/mm; D) 0,5 j A/mm; E) 2,5 i A/mm;

5- Uma lâmina fina e comprida de 2 mm de largura se estende ao longo do eixo x. A lâmina está situada no

plano z =0 e os limites laterais da lâmina são y =

por κκκκ= (1-y2) i, onde y é dado em mm e

atravessa a a reta x=0?

A) 1/3 A; B) 2/3 A; C) 1 A; D) 4/3 A; E) 5/3 A;

Tópicos de Eletromagnetismo I – Prof. Antônio Carlos

Aula 11 – Força de Lorentz

Nome:______________________________________________________________________________

Uma particula de carga q < 0 está se movendo com velocidade constante v = vj quando em t = 0 entra em uma região de campo magnético uniforme B = Bi. A força que atua na partícula em t=0 é:

Um próton se move na direção +z após ser acelerado a partir do repouso por uma diferença de potencial V. O próton então passa através de uma região onde há um campo elétrico uniforme E na direção +x e um campo magnetico uniforme na direção +y, mas a trajetória do próton não é afetada pelos campos. Se o

sse repetido usando uma diferença de potencial de 2V, o próton seria:Defletido na direção +x; Defletido na direção -x; Defletido na direção +y; Defletido na direção –y;

Uma corrente de 1 A passa por um fio de 2 mm de diâmetro . O módulo da densidade superficial de

Uma corrente de 0,5 A é transportada na superfície de um lâmina fina e comprida de 5 mm de largura,

conforme ilustrado pela figura abaixo. A densidade linear de corrente κκκκ é:

Uma lâmina fina e comprida de 2 mm de largura se estende ao longo do eixo x. A lâmina está situada no

plano z =0 e os limites laterais da lâmina são y =± 1 mm. A o vetor densidade linear de corrente é dado

, onde y é dado em mm e κ em A/mm. Qual a corrente I, ( ∫=C

nI ˆ.(rκ

29

Nome:______________________________________________________________________________

quando em t = 0 entra em A força que atua na partícula em t=0 é:

repouso por uma diferença de potencial V. O próton então passa através de uma região onde há um campo elétrico uniforme E na direção +x e um campo magnetico uniforme na direção +y, mas a trajetória do próton não é afetada pelos campos. Se o

sse repetido usando uma diferença de potencial de 2V, o próton seria:

módulo da densidade superficial de

Uma corrente de 0,5 A é transportada na superfície de um lâmina fina e comprida de 5 mm de largura,

Uma lâmina fina e comprida de 2 mm de largura se estende ao longo do eixo x. A lâmina está situada no

1 mm. A o vetor densidade linear de corrente é dado

× ldn )ˆr

) que

Page 30: Topicos de Eletromagnetismo I

30 FIW 591 – Tópicos de Eletromagnetismo I – Prof. Antônio Carlos

6- Um fio fino transporta uma densidade linear de carga λ = 2 C/m com velocidade v = 10-2

m/s. A corrente que atravessa o fio é: A) 0,2 mA; B) 2,0 mA; C) 20,0 mA; D) 0,02 mA; E) 0,002 mA;

7- A densidade de corrente que atravessa (entra) uma superfície fechada S é dada por j= xi (A/m2). A

densidade volumar de carga ρ que se acumula por unidade de tempo no volume definido por S é: A) 0,1 C.m

-3.s

-1;

B) 1,0 C.m-3

.s-1

; C) 10 C.m

-3.s

-1;

D) 100 C.m-3

.s-1

; E) 0,01 C.m

-3.s

-1;

Page 31: Topicos de Eletromagnetismo I

FIW 591 – Tópicos de Eletromagnetismo I

1- Uma partícula de carga q entra em uma

região de campo magnético uniforme

(apontando para dentro da folha). O campo

deflete a partícula de uma distância

conforme mostrado na figura. A carga é

positiva ou negativa? Em termos de

encontre o momento da partícula.

2- Uma trajetória exótica ocorre se uma partícula carregada está sujeita a ação de um campo elétrico

perpendicular a um campo magnético. Suponha que

trajetória de uma partícula que parte da origem com velocidade:

A) v (0) = (E/B)j

B) v (0) = (E/2B)j

C) v (0) = (E/B)(j+k)

3- A superfície carregada r =a (

gira em torno do eixo z com velocidade angular

assim gerada, usando a (a)

Resp(a) e (b) j = σoωasenθ

4- Uma corrente I flui por um fio de raio a. (A) Se ela estiver distribuída uniformemente sobre a superfície,

qual é a densidade de corrente

inversamente proporcional à distância do ei

5- Uma superfície cônica tem uma distribuição de cargas uniforme, com densidade

eixo de simetria com velocidade angular

corrente; (b) a corrente gerada pela rotação do cone. Resp (a)

Tópicos de Eletromagnetismo I – Prof. Antônio Carlos

Aula 11 – Para Casa

Uma partícula de carga q entra em uma

região de campo magnético uniforme B

(apontando para dentro da folha). O campo

deflete a partícula de uma distância d,

conforme mostrado na figura. A carga é

positiva ou negativa? Em termos de a, d, B e q

mento da partícula.

Uma trajetória exótica ocorre se uma partícula carregada está sujeita a ação de um campo elétrico

perpendicular a um campo magnético. Suponha que B aponta na direção x e E na direção z. Encontre a

trajetória de uma partícula que parte da origem com velocidade:

A superfície carregada r =a ( em coordenadas esféricas) com densidade volumar de carga

gira em torno do eixo z com velocidade angular ωωωω=ωz. Determine a densidade superficial da corrente

(a) densidade volumar de cargas; (b) a densidade linear da corrente superficial.

θδ(r-a)φφφφ.

Uma corrente I flui por um fio de raio a. (A) Se ela estiver distribuída uniformemente sobre a superfície,

qual é a densidade de corrente κ? (B) Se ela estiver distribuída de forma que a corrente volumétrica seja

inversamente proporcional à distância do eixo, quanto vale J?

Uma superfície cônica tem uma distribuição de cargas uniforme, com densidade σo e gira em torno do seu

eixo de simetria com velocidade angular ωωωω=ωz. Determine (a) a expressão da densidade linear de

corrente; (b) a corrente gerada pela rotação do cone. Resp (a) κκκκ = σoω(a/h)zφφφφ. (b) I=(1/2)

31

Uma trajetória exótica ocorre se uma partícula carregada está sujeita a ação de um campo elétrico

na direção z. Encontre a

com densidade volumar de carga ρ(r) = σoδ(r-a),

. Determine a densidade superficial da corrente

; (b) a densidade linear da corrente superficial.

Uma corrente I flui por um fio de raio a. (A) Se ela estiver distribuída uniformemente sobre a superfície,

? (B) Se ela estiver distribuída de forma que a corrente volumétrica seja

e gira em torno do seu

. Determine (a) a expressão da densidade linear de

(b) I=(1/2) σoωa(a2 + h

2)

1/2

Page 32: Topicos de Eletromagnetismo I

32 FIW 591 – Tópicos de Eletromagnetismo I – Prof. Antônio Carlos

6- Um fio de 100 cm de comprimento transporta uma corrente de 1,0 A em uma região onde um

campo magnético uniforme de 100 T na direção x. Calcule a força magnética sobre o fio se θ= 45

o é o ângulo entre o fio e a direção x.

A- ( ) 70,7zNB- ( ) 141,4 zNC- ( ) -141,4 zN D- ( ) -70,7ZN E- ( ) 0

Page 33: Topicos de Eletromagnetismo I

FIW 591 – Tópicos de Eletromagnetismo I

Nome:______________________________________________________________________________

1- Duas cargas q e Q se movem com velocidades não nulas com

sobre q exercida por Q é

a) Perpendicular à velocidade de q e depende somente da velocidade de Q;

b) Perpendicular à velocidade de q e depende tanto da velocidade de Q quanto da velocidade de q;

c) Perpendicular à velocidade de Q e depende somente da velocidade de q.

d) Perpendicular à velocidade de Q e depende tanto da velocidade de q quanto da velocidade de Q;

9- Duas cargas positivas q1 e q

carga q1 devido ao campo magnético produzido por q

a) ( ) Entrando na página;b) ( ) Saindo da página;c) ( ) para cima;d) ( ) para baixo;

10- Ainda sobre a questão anterior: qual a direção e o sentido da força sobre a carga q

produzido por q1?

a- ( ) Entrando na página;

b- ( ) Saindo da página;

c- ( ) para cima;

d- ( ) para baixo;

11- Um fio longo e reto situa

movem no sentido de x positivo

sobre o semi-eixo negativo y, aponta em qual direção?

a) +x b)-x c) +y

Tópicos de Eletromagnetismo I – Prof. Antônio Carlos

Aula 12 – Lei de Biot-Savart

Nome:______________________________________________________________________________

Duas cargas q e Q se movem com velocidades não nulas com respeito a um referencial fixo. A força magnética

Perpendicular à velocidade de q e depende somente da velocidade de Q;

Perpendicular à velocidade de q e depende tanto da velocidade de Q quanto da velocidade de q;

à velocidade de Q e depende somente da velocidade de q.

Perpendicular à velocidade de Q e depende tanto da velocidade de q quanto da velocidade de Q;

e q2 estão se movendo para a direita. Qual a direção e o sentido da força sobre

devido ao campo magnético produzido por q2?

( ) Entrando na página; ( ) Saindo da página; ( ) para cima; ( ) para baixo;

Ainda sobre a questão anterior: qual a direção e o sentido da força sobre a carga q2 devido ao campo

( ) Entrando na página;

( ) Saindo da página;

Um fio longo e reto situa-se ao longo do eixo-x e transporta uma corrente de elétrons que se

movem no sentido de x positivo. O campo magnético devido a essa corrente, em um ponto P

eixo negativo y, aponta em qual direção?

c) +y d) –y e)+z f)-z

33

Nome:______________________________________________________________________________

respeito a um referencial fixo. A força magnética

Perpendicular à velocidade de q e depende tanto da velocidade de Q quanto da velocidade de q;

Perpendicular à velocidade de Q e depende tanto da velocidade de q quanto da velocidade de Q;

estão se movendo para a direita. Qual a direção e o sentido da força sobre a

devido ao campo

x e transporta uma corrente de elétrons que se

. O campo magnético devido a essa corrente, em um ponto P

Page 34: Topicos de Eletromagnetismo I

FIW 591 – Tópicos de Eletromagnetismo I

Aula

Nome:______________________________________________________________________________

1- Qual o valor de ∫C

na figura ao lado?

A) ( ) Iµo;

B) ( ) -Iµo;

C) ( ) -2Iµo;

D) ( ) 2Iµo;

2- A figura mostra três correntes de mesmo valor

absoluto i (duas paralelas e uma antiparalela) e quatro

amperianas. Coloque as amperianas em ordem de

acordo com o valor absoluto do fluxo magnético,

começando pelo maior

3- Um fio longo e reto conduz uma

10 A. O valor do campo magnético em um ponto A

distando R = 0,5 m do fio vale, em Tesla (N/A.m),

lembrando que µo

(a) 16×10-6 k

(b) 4×10-6 r

(c) 4×10-6 ϕϕϕϕ

(d) -4×10-6 ϕϕϕϕ

4- Um condutor muito longo tem uma seção transversal quadrada e

seção transversal quadrada. A corrente é uniformemente distribuída ao logo da seção transversal do

condutor. O campo magnético na cavidade é nulo? Justifique a sua resposta.

Este tópico apresenta alto índice de erros

avaliações!

Tópicos de Eletromagnetismo I – Prof. Antônio Carlos

Aula 13 – A divergência e o rotacional de B

Nome:______________________________________________________________________________

∫ ldBrr

. para o caminho mostrado

?

A figura mostra três correntes de mesmo valor

absoluto i (duas paralelas e uma antiparalela) e quatro

amperianas. Coloque as amperianas em ordem de

acordo com o valor absoluto do fluxo magnético,

começando pelo maior.

Um fio longo e reto conduz uma corrente elétrica de

10 A. O valor do campo magnético em um ponto A

,5 m do fio vale, em Tesla (N/A.m),

o =4π×10-7 N/A2

Um condutor muito longo tem uma seção transversal quadrada e contém uma cavidade coaxial com

seção transversal quadrada. A corrente é uniformemente distribuída ao logo da seção transversal do

condutor. O campo magnético na cavidade é nulo? Justifique a sua resposta.

Este tópico apresenta alto índice de erros nas

34

Nome:______________________________________________________________________________

contém uma cavidade coaxial com

seção transversal quadrada. A corrente é uniformemente distribuída ao logo da seção transversal do

Page 35: Topicos de Eletromagnetismo I

35 FIW 591 – Tópicos de Eletromagnetismo I – Prof. Antônio Carlos

Aula 13 – para casa

1-Um fio cilíndrico longo de raio a conduz uma corrente uniformemente distribuída I.

Determine o módulo do campo magnético produzido pela corrente a uma distância a) r>a e b)

r<a;

2- Em uma certa região existe uma densidade de corrente uniforme J no sentido positivo do

eixo z. Determine o valor da integral de caminho do campo magnético quando a integral é

calculada ao longo de três segmentos de reta, de (4d,0,0) →(4d,3d,0)→ (0,0,0)→(4d,0,0).

3- Um cilindro longo condutor oco de raio interno a e raio externo b conduz uma corrente ao

longo de seu eixo de simetria. O módulo da densidade de corrente na seção reta é dado por

J=cr2 . Qual é o vetor campo magnético B em todos os pontos do espaço?

Page 36: Topicos de Eletromagnetismo I

36 FIW 591 – Tópicos de Eletromagnetismo I – Prof. Antônio Carlos

Aula 14 – Potencial Vetor Magnético

Nome:______________________________________________________________________________

1- Se o potencial vetor é dado por A=-yi+xk, então o campo B é:

A) ( ) B = 2k;

B) ( ) B = -2k;

C) ( ) B = -j+k;

D) ( ) B = -1k;

2- Se o potencial vetor é dado por A=-(1/4)µoJoρ2k, (em coordenadas cilíndricas) onde Ao é uma

constante, então a densidade de corrente J é:

A) ( ) J = Jo k;

B) ( ) J = -Jo/4 k;

C) ( ) J = -Jo k;

D) ( ) J = 0

3- Num meio uniforme de permeabilidade µ, uma espira circular de raio a, com eixo OZ e centro na origem, conduz uma corrente I no sentido do unitário azimutal φ . O momento magnético da

espira vale:

A) ( ) m = πa2Ik

B) ( ) m = -πa2Ik

C) ( ) m = 2πa2Ik

D) ( ) m = -2πa2Ik

4- Repita o item anterior usando a expressão ∫×

=C

ldrIm

2

''rr

r

Page 37: Topicos de Eletromagnetismo I

37 FIW 591 – Tópicos de Eletromagnetismo I – Prof. Antônio Carlos

5- Vamos obter uma expressão para o potencial vetor de um dipolo magnético. Vamos trabalhar a partir da equação

∫ −=

Crr

ldIrA

'

'

4)( rr

rr

πµ . O termo

'

1

rrrr

pode ser

escrito como:

A) ( )22 ''.2

1

rrrr +−rr

B) ( )

( )222 ''.2

1

rrrr +−rr

C) ( )22 ''.2

1

rrrr +−rr

D) ( ) 22 ''.2

1

rrrr ++rr

6- A partir da sua resposta do item anterior, fazendo r >> r’, podemos afirmar que o termo

'

1

rrrr

− é aproximadamente igual a:

A) ( )

r

rrr '.21

11rr

B) ( )

r

rrr '.21

11rr

+

C) ( )

2

'.21

11

r

rrrrr

D) ( )

r

rrr '.21

112 rr

7- Podemos desenvolver a raiz quadrada usando uma expressão válida quando x<<1,

(1+x)n≅1+nx. A partir das respostas anteriores,

'

1

rrrr

é aproximadamente igual a:

A) ( ) 2

.ˆ1.ˆ1

1

'

1

r

rr

rr

rr

rrr

rr

rr +=

+≈−

B) ( ) 2

.ˆ1.ˆ1

1

'

1

r

rr

rr

rr

rrr

rr

rr −=

−≈−

C) ( ) 322

.1.1

1

'

1

r

rr

rr

rr

rrr

rr

rr +=

+≈−

8- A partir dos itens anteriores, verifique que o potencial vetor pode ser escrito como

( )

+= ∫∫

CC

ldrrr

ldr

IrA ''.1

'1

4)(

2

rrrr

πµ . O

primeiro termo é nulo. Você pode ver porquê? Vamos trabalhar com o segundo termo. O

integrando ( ) ''. ldrrrr pode ser escrito como:

A) ( ) ( ) ( ) ( ) 'ˆ'.ˆ'''. rrldrldrldrrrrrrrr

+××=

B) ( ) ( ) ( ) ( )rrldrldrldrrrrrrrrˆ'.ˆ''''. +××=

C) ( ) ( ) ( ) ( )rrldrldrldrrrrrrrrˆ'.ˆ'''. +××=

D) ( ) ( ) ( ) ( ) 'ˆ'.ˆ''''. rrldrldrldrrrrrrrr

+××=

(dica: (u××××v)×w=(u.w)v-(v.w)u 9- A diferencial d nas variáveis com linha é

expressa como

( )[ ] ( ) ( )[ ]''.ˆ''.''.ˆ rdrrrrdrrrrdrrrrrr

+= , onde

'' ldrdrr

= . Somando membro a membro esta

expressão com a resposta do item 6, encontramos ( ) [ ] rldrrrrdldrr ˆ)''(')'.(

2

1''. ××+=

rrrrrr .

Quando substituímos esta expressão no segundo termo da integral do item 8, a primeira parcela desaparece, uma vez que é a circulação de uma diferencial exata. A segunda parcela dá:

A) ( ) r

ldrI

rrA

C

ˆ2

''1

4)(

×= ∫

rrr

πµ

B) ( ) r

ldrI

rrA

C

ˆ2

''1

4)(

×= ∫

rrr

πµ

C) ( ) r

ldrI

rrA

C

ˆ2

''1

4)( ×

×= ∫

rrr

πµ

D) ( ) 'ˆ

2

'1

4)(

2r

ldrI

rrA

C

×

×= ∫

rrr

πµ

Finalmente, note que o termo entre colchetes da resposta do item anterior é justamente o momento magnético m. Então:

2

ˆ

4)(

r

rmrA

×=

rr

πµ . Parabéns! Você chegou lá!

Page 38: Topicos de Eletromagnetismo I

38 FIW 591 – Tópicos de Eletromagnetismo I – Prof. Antônio Carlos

Complemento da aula 14 – obtendo o potencial vetor a partir de J

Partindo da lei de Biot-Savart ∫∫∫ −

−×=

V

o dVrr

rrrJB

3'

')'(

4rr

rrrrr

πµ

(1).

Mas 3'

'

'

1

rr

rr

rrrr

rr

rr

r

−−=

−∇ , de modo que podemos escrever

∫∫∫

−∇×−=

V

o dVrr

rJB'

1)'(

4rr

rrrr

πµ

(2).

Usando que vfvfvfrrrrrr

×∇+×∇=×∇ )( , obtemos

( ))'('

1)'(

'

1

'

)'(rJ

rrrJ

rrrr

rJ rrr

rrrr

rr

r

rr

rrr

×∇−

−∇=

−×∇

Como o vetor densidade de corrente depende das variáveis com linha e o rotacional atua nas

coordenadas com linha, o segundo termo da equação acima é nulo.

−∇×−=

−×∇

'

1

' rrJ

rr

Jrr

rr

rr

rr

(3). Substituindo a Eq. (3) em (2): ∫∫∫

−×∇=

V

o dVrr

JB

'4rr

rrr

πµ

. Como o operador nabla não atua nas variáveis com linha:

−×∇= ∫∫∫

V

o dVrr

rJB

'

)'(

4rr

rrrr

πµ

.

Comparando com ABrrr

×∇= , encontramos finalmente: ∫∫∫

−=

V

o dVrr

rJA

'

)'(

4rr

rrr

πµ

PARA CASA: Probls. 5.33-5.37, 5.55-5.61

Page 39: Topicos de Eletromagnetismo I

FIW 591 – Tópicos de Eletromagnetismo I

Nome:______________________________________________________________________________

1- Uma corrente pequena, porém mensurável de

número de portadores de carga por unidade de volume é

uniforme. A densidade de corrente é

A) ( ) j=I/πd2;

B) ( ) j=4I/πd2;

C) ( ) j=I/2πd2;

D) ( ) j=I/d2;

2- Sobre o item anterior, a velocidade de deriva é:

A) ( ) I/ρπd2;

B) ( ) 4I/ρπd2;

C) ( ) I/ρ2πd2;

D) ( ) I/ρd2;

3- O sentido da f.e.m suprida por uma bateria depende do sentido da corrente que flui através

desta bateria? ( ) SIM ( )NÃO

4- Um resistor cilíndrico cujo corte transversal tem área A e comprimento L é feito de material

com condutividade σ. Se o potencial é constante nas duas extremidades e a diferença de

potencial é V, que corrente está pa

5- Uma bateria de fem E

Se você quiser fornecer o máximo possível de potência para a resistência de carga, que

resistência R deve escolher?

6-Prova de Mestrado (2012)

Tópicos de Eletromagnetismo I – Prof. Antônio Carlos

Aula 15 – Força eletromotriz

Nome:______________________________________________________________________________

Uma corrente pequena, porém mensurável de I atravessa um fio de cobre de diâmetro

número de portadores de carga por unidade de volume é ρ. Supondo que a corrente é

uniforme. A densidade de corrente é

Sobre o item anterior, a velocidade de deriva é:

m suprida por uma bateria depende do sentido da corrente que flui através

desta bateria? ( ) SIM ( )NÃO

Um resistor cilíndrico cujo corte transversal tem área A e comprimento L é feito de material

. Se o potencial é constante nas duas extremidades e a diferença de

potencial é V, que corrente está passando? Resp: I=σAV/L

e resistência interna r está ligada a uma resistência de carga variável R.

Se você quiser fornecer o máximo possível de potência para a resistência de carga, que

resistência R deve escolher?

39

Nome:______________________________________________________________________________

atravessa um fio de cobre de diâmetro d. O

. Supondo que a corrente é

m suprida por uma bateria depende do sentido da corrente que flui através

Um resistor cilíndrico cujo corte transversal tem área A e comprimento L é feito de material

. Se o potencial é constante nas duas extremidades e a diferença de

e resistência interna r está ligada a uma resistência de carga variável R.

Se você quiser fornecer o máximo possível de potência para a resistência de carga, que

Page 40: Topicos de Eletromagnetismo I

40 FIW 591 – Tópicos de Eletromagnetismo I – Prof. Antônio Carlos

Aula 16 – Indução eletromagnética

Nome:______________________________________________________________________________

1- O fluxo magnético através de uma espira de fio varia de ∆Φ durante o intervalo de tempo ∆t. A

mudança de fluxo ∆Φ é proporcional: A) ( ) A corrente no fio; B) ( ) A resistência do fio; C) ( ) A carga resultante que flui através de qualquer seção do fio. D) ( ) A d.d.p entre quaisquer dois pontos fixos do fio;

2- Considere um fio horizontal muito longo por onde flui uma corrente estacionária i (para a

direita) e uma espira retangular com dois de seus lados paralelos ao fio. A espira e o fio estão no mesmo plano, como indica a figura. A espira é colocada em movimento de translação, afastando-se do fio com velocidade de módulo v.

A respeito do sentido da corrente induzida na espira e da força exercida sobre a espira, dada por

∫ ×= BlidFrrr

, podemos afirmar

que:

(a) o sentido é horário e a força é atrativa; (b) o sentido é horário e a força é repulsiva; (c) o sentido é anti-horário e a força é atrativa; (d) o sentido é anti-horário e a força é repulsiva; (e) não surge corrente induzida e, portanto, a força é nula.

3- Um campo magnético variável B (apontando perpendicularmente para fora do papel, como na figura) está confinado ao interior de um tubo condutor. Se o fluxo do campo magnético aumenta com o tempo, surge uma corrente elétrica no cilindro condutor.

a) Na mesma direção e sentido oposto ao do campo magnético;

b) Na mesma direção e mesmo sentido do campo magnético;

c) No plano perpendicular ao campo magnético e circulando no sentido horário;

d) No plano perpendicular ao campo magnético e circulando no sentido anti-horário;

Page 41: Topicos de Eletromagnetismo I

41 FIW 591 – Tópicos de Eletromagnetismo I – Prof. Antônio Carlos

Aula 16 – para casa

1- Prova de acesso ao Mestrado Profissional em Ensino de Física (UFRJ)

Problemas do livro texto: 7.12 até 7.29

Page 42: Topicos de Eletromagnetismo I

42 FIW 591 – Tópicos de Eletromagnetismo I – Prof. Antônio Carlos

Aula 17 – Equações de Maxwell I

Nome:______________________________________________________________________________

Considere as equações de Maxwell:

A) . ρ Lei de Gauss da eletricidade

B) . 0 Lei de Gauss do Magnetismo

C)

Lei de Faraday da Indução

D) ! "

# Lei de Ampère generalizada

1- Qual das equações de Maxwell implica na não existência do monopólo magnético?

A) ( )B) ( )C)( ) D) ( )

2- Qual das equações de Maxwell nos diz sobre a existência do monopólo elétrico?

A) ( )B) ( )C)( ) D) ( )

3- Qual das equações de Maxwell nos diz que um campo magnético que varia induz um campo

elétrico?

A) ( )B) ( )C)( ) D) ( )

4- Qual das equações de Maxwell nos diz que um campo elétrico que varia induz um campo

elétrico?

A) ( )B) ( )C)( ) D) ( )

5- Verdadeiro ou falso:

A) ( ) a corrente de deslocamento tem unidade diferente da corrente de condução

B) ( ) a corrente de deslocamento apenas existe se o campo elétrico na região está variado

no tempo;

C) ( ) Em um circuito LC oscilante, não existe corrente de deslocamento entre as placas do

capacitor quando ele está momentaneamente completo de carga

D) ( ) Em um circuito LC oscilante, não existe corrente de deslocamento entre as placas do

capacitor quando ele está momentaneamente vazio

E) ( ) As equações de Maxwell se aplicam apenas a campos elétricos e magnéticos que são

constantes no tempo;

6- O campo magnético em uma região do espaço é dado por B = kt2i para -2s ≤ t ≤ 2s. Qual o

sentido do campo elétrico induzido quando t= 0?

A) Paralelo ao eixo x;

B) Paralelo ao eixo y;

C) O campo elétrico está disposto em círculos centrados no eixo x;

D) Não há campo elétrico quando t= 0s;

7- As figuras abaixo mostram, em várias situações, o campo elétrico e o campo magnético

induzido. Determine em cada caso se o módulo do campo elétrico está aumentando ou

diminuindo.

Page 43: Topicos de Eletromagnetismo I

FIW 591 – Tópicos de Eletromagnetismo I

A) B)

1- prova de acesso à pós-graduação em física (UNIPÓS

Tópicos de Eletromagnetismo I – Prof. Antônio Carlos

C) D)

Aula 17 -para casa

graduação em física (UNIPÓS -2011)

43

Page 44: Topicos de Eletromagnetismo I

44 FIW 591 – Tópicos de Eletromagnetismo I – Prof. Antônio Carlos

Aula 18 – Equações de Maxwell na matéria

Nome:______________________________________________________________________________ (dielétricos) Um capacitor de placas paralelas de área A e separação d é preenchido com duas placas dielétricas. A

placa dielétrica superior possui uma espessura de d e permissividade ε1=6εo e a placa inferior possui uma espessura

de 2d com ε2=12εo. Se uma d.d.p V é aplicada no capacitor, encontre os valores das seguintes grandezas em cada dielétrico:

1) a capacitância

A) ( ) εoA /2d;

B) ( ) εoA /d;

C) ( ) 2εoA /d;

D) ( ) 4εoA /d;

E) ( ) 6εoA /d; 1) A carga nas placas do capacitor

A) ( ) εoA V/2d;

B) ( ) εoAV/d;

C) ( ) 2εoAV/d;

D) ( ) 4εoAV/d;

E) ( ) 6εoAV/d;

2) A densidade superficial de carga σ

A) ( ) εoV/2d;

B) ( ) εoV/d;

C) ( ) 2εoV/d;

D) ( ) 4εoV/d;

E) ( ) 6εoV/d; 3) O vetor deslocamento D em cada um dos dielétricos

A) ( ) εoV/2d;

B) ( ) εoV/d;

C) ( ) 2εoV/d;

D) ( ) 4εoV/d;

E) ( ) 6εoV/d; 4) O campo elétrico em cada um dos dielétricos

A) ( ) E1 = V/12d; E2 =V/24d B) ( ) E1 = V/6d; E2 =V/12d C) ( ) E1 = V/3d; E2 =V/6d D) ( ) E1 = 2V/3d; E2 =V/3d E) ( ) E1 = V/d; E2 =V/2d

5) O vetor de polarização em cada um dos dielétricos

A) ( ) P1 = 10εoV/3d; P2 = 11εoV/3d

B) ( ) P1 = 11εoV/3d; P2 = 10εoV/3d

C) ( ) P1 = 10εoV/d; P2 = 11εoV/d

D) ( ) P1 = εoV/3d; P2 = εoV/d

E) ( ) P1 = 2εoV/3d; P2 = εoV/3d

6) (meios magnéticos) Um meio ferromagnético de grande extensão possui B = 2Tz. Se µ =175µo,

encontre H e M.

7) (condições de contorno). Mostre que, na fronteira descarregada que separa dois meios de

permissividades ε1 e ε2, vale a lei da refração do campo elétrico. ε1 cotgθ1 =ε2 cotgθ2

8) (condições de contorno) Mostre que, na fronteira descarregada que separa dois meios de

permeabilidades µ1 e µ2, vale a lei da refração do campo magnético. µ1 cotgθ1 =µ2 cotgθ2

Page 45: Topicos de Eletromagnetismo I

45 FIW 591 – Tópicos de Eletromagnetismo I – Prof. Antônio Carlos

Aula 19 – a conservação da carga e da energia

Nome:______________________________________________________________________________

1 – (conservação da carga) A densidade de corrente que sai de um volume é dada, em coordenadas

esféricas, por rjj oˆ=

r, onde jo é uma constante. A taxa de variação da densidade volumar de carga

∂ρ/∂t é (vide no formulário a expressão para o operador nabla em coordenadas esféricas):

A) ( ) 0;

B) ( ) -jo/r;

C) ( ) +2jo/r;

D) ( ) -2jo/r;

2- (conservação da energia) Em uma dada região do espaço linear e isotrópico há um campo

eletromagnético dependente do tempo. Se o campo elétrico aponta da direção positiva do eixo

x e o campo magnético aponta na direção negativa do eixo y, o vetor de Poynting aponta:

A) ( ) Na direção +z;

B) ( ) Na direção –z;

C) ( ) Na direção +x;

D) ( ) Na direção –y;

3- Em um meio de permissividade elétrica ε e campo magnético nulo, a densidade volumar de

energia é dada por:

A) ( ) u = E2/2ε;

B) ( ) u = εE2/2;

C) ( ) u = E2/2;

D) ( ) u = εE2;

4- Em um meio de permeabilidade magnética µ e campo elétrico nulo, a densidade volumar de

energia eletromagnética é dada por:

A) ( ) u = B2/2µ;

B) ( ) u = µB2/2;

C) ( ) u = B2/2;

D) ( ) u = µB2;

Page 46: Topicos de Eletromagnetismo I

46 FIW 591 – Tópicos de Eletromagnetismo I – Prof. Antônio Carlos

Aula 20 – a conservação do momento

Nome:______________________________________________________________________________

1- Um fluido em condições estáticas não está sujeito a tensões de cisalhamento. Se a pressão em

um ponto do fluido é po, o tensor de tensões nestas condições é expresso por :

A) ( )

o

o

o

p

p

p

00

00

00

B) ( )

ooo

ooo

ooo

ppp

ppp

ppp

C) ( )

0

0

0

oo

oo

oo

pp

pp

pp

2- Um capacitor é composto por duas placas paralelas de área A e separação d prenchida por um

dielétrico de permissividade elétrica ε. Aplica-se uma d.d.p V entre as placas do capacitor. A

força que uma placa exerce sobre a outra é (em módulo):

A) ( ) A(ε/2)(V/d)2

B) ( ) Aε(V/d)2

C) ( ) 2Aε(V/d)2

D) ( ) A(ε/4)(V/d)2

3- No caso do capacitor do exemplo anterior, se o sinal das placas for alterado, a tensão na

direção do campo :

A) ( ) não se altera;

B) ( ) altera o sinal mas não o módulo;

C) ( ) altera o módulo e o sinal;

4- Se o campo elétrico é dado por E=(Eo. 0,0), o tensor de tensões de Mawell é dado por

A) ( )

2

0

2

0

2

0

0

00

00

00

2E

E

E

ε

B) ( )

2

0

2

0

2

0

0

00

00

00

2E

E

E

ε

C) ( ) ( )2

0

2

0

2

0

0

00

00

00

E

E

E

ε

D) ( ) ( )2

0

2

0

2

0

0

00

00

00

E

E

E

ε−

5- No caso do exercício anterior, 0 fluxo de momento (momento/área) na direção y é:

A) ( ) )0,2

,0(2

00ETy

ε=−

r

B) ( ) )0,0,2

(2

00ETy

ε=−

r

C) ( ) ),0,0( 2

00ETy ε=−r

Page 47: Topicos de Eletromagnetismo I

FIW 591 – Tópicos de Eletromagnetismo I

É extremamente valioso para o ensino de física o fato que algumas grandezas, a princípio distintas,

possam ser discutidas igualmente. Algumas grandezas físicas que se comportam analogamente aos

fluidos. Elas são chamadas de quantidades tipo substância. Entre elas e

elétrica, momento, momento angular, entropia, etc... Cada uma delas pode ser imaginada como um tipo

de substância. Uma indicação de que uma quantidade X ( X pode ser densidade de carga, densidade de

energia, densidade de momento

da continuidade:

XX

XX

Idt

dX

ou

wjt

X

Σ+=

+⋅∇=∂∂ rr

Ou na forma integral:

∫∫ ∫∫∫ =+⋅S V

X XdVdt

dSdj 0rr

Que se aplica a uma dada região do espaço de volume V. A quantidade dX/dt (

de variação temporal de X dentro da região,

unidade de tempo no interior daquela região. I

da superfície que limita a região. Assim, há duas causas para a mudança no valor de X dentro do volume

V: a criação ou destruição de X dentro da região e uma corrente de X através da superfície S que limita

V.

Para algumas grandezas, o termo

dentro de V quando uma corrente flui através da superfície S. Estas grandezas são chamadas

conservadas, por exemplo: carga elétrica (dQ/dt+I=0), energia (dE/dt=P), momento (d

representa a corrente elétrica, P a corrente de energia (potência), e

Uma grandeza tipo sustância não deve necessariamente ser conservada. Por obedecerem à equação da

continuidade, grandezas tipo substância possuem algumas propriedades que fazem ser

fácil de lidar com estas grandezas:

I) O valor da quantidade tipo substância refere

II) Todo grandeza tipo substância possui outra grandeza associada a ela que pode ser

interpretada como uma corrente.

III) Quantidade tipo su

IV) Correntes também são aditivas;

Considere a figura abaixo: podemos descrever esta situação dizendo que um trabalho está sendo

realizado sobre as placas do capacitor, aumentando sua energia potencial. A mesma situação pode ser

descrita utilizando o caráter tipo substância da energia: Energia flui das mãos através das cordas para o

campo do capacitor.

Tópicos de Eletromagnetismo I – Prof. Antônio Carlos

mente valioso para o ensino de física o fato que algumas grandezas, a princípio distintas,

possam ser discutidas igualmente. Algumas grandezas físicas que se comportam analogamente aos

fluidos. Elas são chamadas de quantidades tipo substância. Entre elas estão: massa, energia, carga

elétrica, momento, momento angular, entropia, etc... Cada uma delas pode ser imaginada como um tipo

de substância. Uma indicação de que uma quantidade X ( X pode ser densidade de carga, densidade de

energia, densidade de momento, etc...) se comporta como substância é quando ela obedece à equação

Que se aplica a uma dada região do espaço de volume V. A quantidade dX/dt (∂X/∂t) representa a taxa

de variação temporal de X dentro da região, ΣX (wX), indica o quanto de X é criado ou destruído por

unidade de tempo no interior daquela região. IX (divjx) representa a intensidade de corrente de X através

que limita a região. Assim, há duas causas para a mudança no valor de X dentro do volume

V: a criação ou destruição de X dentro da região e uma corrente de X através da superfície S que limita

Para algumas grandezas, o termo ΣX é sempre nulo. Estas grandezas apenas podem alterar seu valor

dentro de V quando uma corrente flui através da superfície S. Estas grandezas são chamadas

conservadas, por exemplo: carga elétrica (dQ/dt+I=0), energia (dE/dt=P), momento (d

ica, P a corrente de energia (potência), e F a corrente de momento (força).

Uma grandeza tipo sustância não deve necessariamente ser conservada. Por obedecerem à equação da

continuidade, grandezas tipo substância possuem algumas propriedades que fazem ser

fácil de lidar com estas grandezas:

O valor da quantidade tipo substância refere-se a uma região no espaço;

Todo grandeza tipo substância possui outra grandeza associada a ela que pode ser

interpretada como uma corrente.

Quantidade tipo substância são aditivas;

Correntes também são aditivas;

Considere a figura abaixo: podemos descrever esta situação dizendo que um trabalho está sendo

realizado sobre as placas do capacitor, aumentando sua energia potencial. A mesma situação pode ser

utilizando o caráter tipo substância da energia: Energia flui das mãos através das cordas para o

47

mente valioso para o ensino de física o fato que algumas grandezas, a princípio distintas,

possam ser discutidas igualmente. Algumas grandezas físicas que se comportam analogamente aos

stão: massa, energia, carga

elétrica, momento, momento angular, entropia, etc... Cada uma delas pode ser imaginada como um tipo

de substância. Uma indicação de que uma quantidade X ( X pode ser densidade de carga, densidade de

, etc...) se comporta como substância é quando ela obedece à equação

t) representa a taxa

), indica o quanto de X é criado ou destruído por

) representa a intensidade de corrente de X através

que limita a região. Assim, há duas causas para a mudança no valor de X dentro do volume

V: a criação ou destruição de X dentro da região e uma corrente de X através da superfície S que limita

andezas apenas podem alterar seu valor

dentro de V quando uma corrente flui através da superfície S. Estas grandezas são chamadas

conservadas, por exemplo: carga elétrica (dQ/dt+I=0), energia (dE/dt=P), momento (dp/dt=F), onde I

a corrente de momento (força).

Uma grandeza tipo sustância não deve necessariamente ser conservada. Por obedecerem à equação da

continuidade, grandezas tipo substância possuem algumas propriedades que fazem ser relativamente

se a uma região no espaço;

Todo grandeza tipo substância possui outra grandeza associada a ela que pode ser

Considere a figura abaixo: podemos descrever esta situação dizendo que um trabalho está sendo

realizado sobre as placas do capacitor, aumentando sua energia potencial. A mesma situação pode ser

utilizando o caráter tipo substância da energia: Energia flui das mãos através das cordas para o

Page 48: Topicos de Eletromagnetismo I

FIW 591 – Tópicos de Eletromagnetismo I

O momento é mais do que simplesmente m

como mv . Por exemplo, onda eletromagnética:

O Ensino tradicional de física nem sempre faz

geralmente derivadas a partir de

que estes também são semelhantes à

6- Dado o vetor de Poyinting (

A) ( ) µεS;

B) ( ) -µεS;

C) ( ) εS;

D) ( ) µS;

Grandeza X

Carga elétrica ρ (carga/volume)

Energia

eletromagnética

µ (energia/volume)

momento Pem

(momento/volume)

Densidade de momento angular:

elemento de carga em relação à origem.

O momento angular do campo:

Referência: The Karlsruhe Physics Course

Tópicos de Eletromagnetismo I – Prof. Antônio Carlos

O momento é mais do que simplesmente mv. Há sistemas cujo momentos não podem ser descritos

. Por exemplo, onda eletromagnética: BDem

rr×=℘

(densidade de momento)

nem sempre faz uso de estas vantagens. A energia,

a partir de outros quantidades. Isto faz com que seja mais difícil de compreender

semelhantes à quantidades tipo substância.

Dado o vetor de Poyinting ( S=E×H), a densidade de momento eletromagnético é:

Grandeza X Corrente de X Equação

(carga/volume)

j=ρv (carga/área. tempo) +⋅∇ j

r

(energia/volume) S=µv

(energia/área.tempo) S⋅∇rr

(momento/volume) Tt

−(momento/área.tempo)

T⋅∇tr

Momento angular

Densidade de momento angular: )( BDrr emem

rrrrrlr

××=℘×≡ , onde r é o vetor posição do

elemento de carga em relação à origem.

∫∫∫=V

emem dVL lrr

The Karlsruhe Physics Course

48

. Há sistemas cujo momentos não podem ser descritos

(densidade de momento)

A energia, e momento são

is difícil de compreender

), a densidade de momento eletromagnético é:

Equação

0=∂∂

+t

ρ

wt

−=∂∂

t

pp mecem

+∂=

)(

é o vetor posição do

Page 49: Topicos de Eletromagnetismo I

49 FIW 591 – Tópicos de Eletromagnetismo I – Prof. Antônio Carlos

Para casa

1- Um capacitor de placas paralelas espaçadas de uma distância d com uma diferença de potencial

V entre elas. Considere que o dielétrico entre as placas é o vácuo. A tensão (força por unidade

de área) paralela às linhas de campo elétrico é

A) ( ) εoV2/d

2; B) ( ) 2εoV

2/d

2; C) ( ) εoV/d; D) ( ) εoV

2/2d

2;

2- Assinale a afirmativa correta

A) ( ) Em cada ponto de um campo elétrico há uma tensão na direção do campo elétrico.

Sua intensidade é εoE2/2;

B) ( ) Em cada ponto de um campo elétrico há uma tensão na direção do campo elétrico.

Sua intensidade é εoE2/2;

C) ( ) Em cada ponto de um campo elétrico há uma tensão na direção do campo elétrico.

Sua intensidade é εoE2/2;

D) ( ) Em cada ponto de um campo elétrico há uma tensão na direção do campo elétrico.

Sua intensidade é εoE2/2;

3- Um campo eletromagnético é representado pelo vetor deslocamento D=Doi e pelo campo

magnético B=Bok. A densidade de momento do campo é:

a) ( ) DoBoj;

b) ( ) -DoBoj;

c) ( ) DoBok;

d) ( ) DoBoi;

4- Se o tensor de Maxwell é dado por

00

020

0

o

o

oo

T

T

TT

T

=t

, o momento transportado pelos

campos na direção x atravessando uma superfície orientada na direção z por unidade de área,

por unidade de tempo (ou tensão eletromagnética - força/área) é:

A) ( ) To;

B) ( ) -To;

C) ( ) 2To;

D) ( ) -2To;

E) ( ) 0;

5- A densidade de momento angular do campo na posição r=roj de um campo eletromagnético

representado pelo vetor deslocamento D=Doj e pelo campo magnético B=Bok é:

A) ( ) roDoBoi;

B) ( ) -roDoBoi;

C) ( ) roDoBoj;

D) ( ) -roDoBok;

E) ( ) roDoBok;

Page 50: Topicos de Eletromagnetismo I

50 FIW 591 – Tópicos de Eletromagnetismo I – Prof. Antônio Carlos

Aula 21 – Ondas eletromagnéticas em uma dimensão

Nome:______________________________________________________________________________

1- Dada a equação de onda 2

2

2

2

tz ∂∂

=∂∂ ψ

µεψ

, qual é a velocidade de propagação da onda?

A) ( ) εµ;

B) ( ) 1/εµ;

C) ( ) (εµ)2 ;

D) ( ) 1/(εµ)2 ;

Dada a função de onda ψ=Asen2π(3x-2t), onde x é dado em metros e t em segundos.

Determine:

2- O comprimento de onda em metros:

A) ( ) 1/3;

B) ( ) 2π/3;

C) ( ) 1;

D) ( ) 1/6;

3- A frequência em Hz:

A) ( ) 3;

B) ( ) 1/π;

C) ( ) 1;

D) ( ) 2;

4- A velocidade de fase em m/s:

A) ( ) 2/3;

B) ( ) 1/3;

C) ( ) 2;

D) ( ) 1;

5- O período em segundos:

A) ( ) 1/3;

B) ( ) π;

C) ( ) 1;

D) ( ) 1/2;

5) A direção de propagação:

A) ( ) +z; B) ( ) -z; C) ( ) +x; D) ( ) –x;

6) Calcule o complexo conjugado de z1 = 2+3i e z2 =2ei2

A) ( ) 2-3i e 2e-2i

;

B) ( ) -2+3i e 2e-2i

;

C) ( ) 2+3i e -2e2i

;

D) ( ) -2-3i e -2e-2i

;

7) Calcule a norma de z=3e-i2

A) ( ) 3; B) ( ) -3; C) ( ) 2; D) ( ) -2;

Page 51: Topicos de Eletromagnetismo I

51 FIW 591 – Tópicos de Eletromagnetismo I – Prof. Antônio Carlos

8) Dado z=3eiπ. Re(z) e Im(z) valem respectivamente:

A) ( ) -3 e 0;

B) ( ) 3 e 0;

C) ( ) 0 e 3;

D) ( ) 0 e -3;

Descreva completamente o estado de polarização de cada uma das seguintes ondas:

9) E =Eo cos(kz-ωt)i-Eo cos(kz-ωt)j A) ( ) linear; B) ( ) circular; C) ( ) eliptica;

10) E =Eo sen(kz-ωt)i-Eo sen(kz-ωt)j A) ( ) linear; B) ( ) circular; C) ( ) eliptica;

11) E =Eo sen(kz-ωt)i+Eo sen(kz-ωt-π/4)j A) ( ) linear; B) ( ) circular; C) ( ) eliptica;

12)E =Eo cos(kz-ωt)i+Eo cos(kz-ωt+π/2)j

A) ( ) linear; B) ( ) circular; C) ( ) eliptica;

Page 52: Topicos de Eletromagnetismo I

52 FIW 591 – Tópicos de Eletromagnetismo I – Prof. Antônio Carlos

Aula 21 – para casa

1- Indique cada um dos números z =x+iy abaixo no plano complexo. Para cada número de o valor

número de sua parte real (ReZ=x) , sua parte imaginária y (Im Z=y), seu módulo r e o valor de θ.

a) 1+i;

b) i-1;

c) 2i;

d) 3;

e) 2i-2;

Você sabia?

Imagens 3D usam luz polarizada para produzir sensação de profundidade. Nossa percepção de

profundidade é devido, em grande parte, a habilidade do olho de enxergar o mundo de um

ângulo ligeiramente diferente. O cérebro funde as duas visões em uma tridimensional. Durante

a filmagem de um filme 3D, duas câmeras gravam a ação de perspectivas ligeiramente

diferentes. Quando o filme é mostrado, dois projetores com filtros polarizadores, são

utilizados.

Page 53: Topicos de Eletromagnetismo I

53 FIW 591 – Tópicos de Eletromagnetismo I – Prof. Antônio Carlos

Aula 22 – Ondas eletromagnéticas no vácuo

Nome:______________________________________________________________________________

1- Falando informalmente, pode-se dizer que as componentes elétrica e magnética de uma

onda eletromagnética progressiva “alimentam-se uma à outra”. O que isto significa?

2- Considere a seguinte configuração de linhas de campo. Ela pode estar relacionada a..

A) ( ) somente ao campo elétrico;

B) ( ) somente ao campo

magnético;

C) ( ) a ambos;

D) ( ) a nenhum dos dois;

3- Considere a seguinte configuração de linhas de campo. Ela pode estar relacionada a..

E) ( ) somente ao campo elétrico;

F) ( ) somente ao campo

magnético;

G) ( ) a ambos;

H) ( ) a nenhum dos dois;

4- Um laser de hélio-neônio emite luz vermelha em uma faixa estreita de comprimentos de

onda em torno de λ, com uma largura de ∆λ. Qual é a largura da luz emitida em unidades

de freqüência?

A) ( ) c∆λ/λ2;

B) ( ) c/λ;

C) ( ) c/λ2;

D) ( ) c∆λ/λ;

5- Em um determinado ponto e em um determinado instante de tempo o campo elétrico de

uma onda onda eletromagnética aponta para o norte quando o campo magnético aponta

para cima. Em que direção a onda eletromagnética está se propagando?

A- ( ) Leste; B- ( ) Oeste; C-( )Sul D-( ) para baixo;

6- O campo elétrico de uma onda plana que viaja na direção do eixo z é E=(Eoxx+Eoy y)sen(ωt-

kz+ϕ). Encontre o campo magnético

A) ( ) B=(-Eoyi+Eox j)cos(ωt-kz+ϕ)/c

B) ( ) B=(Eoxi+Eoy j)sen(ωt-kz+ϕ)/c

C) ( ) B=(-Eoyi+Eox j)sen(ωt-kz+ϕ)/c

D) ( ) B=(Eoxi+Eoy j)cos(ωt-kz+ϕ)/c

E) ( ) B=(-Eoyi-Eoy j)sen(ωt-kz+ϕ)/c

Page 54: Topicos de Eletromagnetismo I

54 FIW 591 – Tópicos de Eletromagnetismo I – Prof. Antônio Carlos

7- Se um feixe de luz vermelha, um feixe de verde e um feixe de luz violeta, todos viajando no

vácuo, têm a mesma intensidade, qual o feixe tem a maior momento?

a) ( ) o de luz vermelha;

b) ( ) o de luz verde;

c) ( ) o de luz violeta;

d) ( ) eles todos têm o mesmo momento;

e) ( ) não se pode determinar a partir destes dados;

8- Ainda sobre o item anterior, qual feixe tem o maior valor de pico (amplitude) para o campo

elétrico?

a) ( ) o de luz vermelha;

b) ( ) o de luz verde;

c) ( ) o de luz violeta;

d) ( ) eles todos têm o mesmo momento;

e) ( ) não se pode determinar a partir destes dados;

9- Duas ondas planas eletromagnéticas senoidas são idênticas, exceto que a onda A tem uma

amplitude do campo elétrico que é três vezes a amplitude da onda B. Como se comparam

suas intensidades?

A) ( ) IA =IB/3;

B) ( ) IA =IB/9;

C) ( ) IA =3IB;

D) ( ) IA =9IB;

Page 55: Topicos de Eletromagnetismo I

55 FIW 591 – Tópicos de Eletromagnetismo I – Prof. Antônio Carlos

Aula 22 – Ondas eletromagnéticas no vácuo (para casa)

Equações úteis (complementam o formulário):

Pressão de radiação (P) e intensidade (I): 2

2

ooE

c

I

c

SP

ε=== , onde Eo é a amplitude do campo

elétrico.

Momento (p) e energia (U) em uma onda eletromagnética: c

Up =

1- Os espelhos usados em um tipo particular de laser refletem 99,99 % da radiação incidente. (a)

Se o laser emite uma potência média de 15 W, qual é a potência média da radiação incidente

em um dos espelhos? b) qual é a força devida à pressão de radiação em um dos espelhos?

2- Um laser pulsado dispara um pulso de 1000 MW com duração de 200 ns em um pequeno

objeto que tem massa de 10,0 mg e está suspenso por uma fina fibra de 4,00 cm de

comprimento. Se a radiação for completamente absorvida pelo objeto, qual é o máximo ângulo

de deflexão deste pêndulo? (Pense no sistema como se fosse um pêndulo balístico e considere

que o pequeno objeto estivesse pendurado verticalmente antes que a radiação o atingisse)

3- A intensidade da luz do Sol ao atingir a Terra é de aproximadamente 1300 W/m2. Se a luz do Sol

atingir um absorvedor perfeito, que pressão irá exercer? E sobre um refletor perfeito? A que

fração da pressão atmosférica isso equivale? (pressão atmosférica ≅ 105 Pa)

4- Exame de acesso à pós-graduação em física (UNIPÓS-2012)

Page 56: Topicos de Eletromagnetismo I

56 FIW 591 – Tópicos de Eletromagnetismo I – Prof. Antônio Carlos

5- Exame de acesso à pós-graduação em física (UNIPÓS-2010)

Page 57: Topicos de Eletromagnetismo I

57 FIW 591 – Tópicos de Eletromagnetismo I – Prof. Antônio Carlos

Aula 23– Ondas eletromagnéticas na matéria

Nome:______________________________________________________________________________

1- Em um meio não dispersivo de impedância Z=2Zo , onde Zo é a impedância do vácuo, a luz se

propaga com velocidade v=c/2. As permissividade elétrica e permeabilidade magnéticas são,

respectivamente:

A) ( ) 4, o

o

µε ;

B) ( )

oo µε 4, ;

C) ( )

o

o µε,

4;

D) ( )

oo µε ,4 ;

2- Os ângulos de incidência e transmissão para um raio incidente no ar (meio 1) em bloco (meio 2)

são 45o e 30

o, respectivamente. O índice de refração do bloco é:

A) ( ) 2 ;

B) ( ) 2

1;

C) ( ) 3 ;

D) ( ) 3

1;

3- Para uma onda eletromagnética que incide normalmente sobre um meio, os coeficientes de

reflexão para as componentes perperdicular e paralela ao plano de incidência do campo

elétrico são, respectivamente:

A) ( ) 1,12

12

ZZ

ZZ

+

B) ( ) 1,12

12 −+

ZZ

ZZ

C) ( ) 1,12

21

ZZ

ZZ

+

D) ( )12

12,1ZZ

ZZ

+

−−

4- Para um meio onde n2=3, o ângulo de incidência para o qual a luz refletida é 100 % polarizada

é:

A) ( ) 30o;

B) ( ) 45o;

C) ( ) 60o;

D) ( ) 80o;

Page 58: Topicos de Eletromagnetismo I

FIW 591 – Tópicos de Eletromagnetismo I

Aula 23 –

1- A impedância intrínseca de um meio é Z = 200

encontre ε e µ para o meio.

2- Mostre que, na fronteira descarregada que separa dois meios de permissividades

lei da refração do campo elétrico:

3- A velocidade da luz na água é menor do que a velocidade da luz no ar. Quando um raio de luz

vermelha, λ=650 nm, move

A- ( ) a freqüência;

B- ( ) o comprimento de onda;

C- ( ) a cor;

D- ( ) itens A e B;

E- ( ) itens A, B e C;

4- Quando o tanque de metal retangular mostrado na figura é preenchido até o topo com um

líquido desconhecido, um observador com os olhos no nível do topo do tanque somente é

capaz de ver o canto E. Determine o índice de refração do líquido.

5- Mostre que a velocidade de grupo pode ser escrita na forma

6- Mostre que a velocidade de grupo pode ser escrita na forma

7- Mostre que a velocidade de grupo pode ser escrita na forma

8- Exemplo de polarização por reflexão. Você seria capaz de dizer em qual das fotos abaixo foi

utilizado um filtro polarizador?

Tópicos de Eletromagnetismo I – Prof. Antônio Carlos

– Ondas eletromagnéticas na matéria (para casa)

A impedância intrínseca de um meio é Z = 200 Ω e a velocidade de fase no meio é v

para o meio.

Mostre que, na fronteira descarregada que separa dois meios de permissividades

lei da refração do campo elétrico: ε1cotg θ1 =ε2cotgθ2.

na água é menor do que a velocidade da luz no ar. Quando um raio de luz

=650 nm, move-se do ar para a água, o que muda?

( ) o comprimento de onda;

Quando o tanque de metal retangular mostrado na figura é preenchido até o topo com um

desconhecido, um observador com os olhos no nível do topo do tanque somente é

capaz de ver o canto E. Determine o índice de refração do líquido.

Mostre que a velocidade de grupo pode ser escrita na forma λd

dvvv fg −=

Mostre que a velocidade de grupo pode ser escrita na forma

+=

ωd

dnn

cvg

Mostre que a velocidade de grupo pode ser escrita na forma

+=λn

c

n

cvg 2

Exemplo de polarização por reflexão. Você seria capaz de dizer em qual das fotos abaixo foi

utilizado um filtro polarizador?

58

e a velocidade de fase no meio é vf =108 m/s

Mostre que, na fronteira descarregada que separa dois meios de permissividades ε1 e ε2, vale a

na água é menor do que a velocidade da luz no ar. Quando um raio de luz

Quando o tanque de metal retangular mostrado na figura é preenchido até o topo com um

desconhecido, um observador com os olhos no nível do topo do tanque somente é

λd

dv f.

ωddn

λd

dn.

Exemplo de polarização por reflexão. Você seria capaz de dizer em qual das fotos abaixo foi

Page 59: Topicos de Eletromagnetismo I

FIW 591 – Tópicos de Eletromagnetismo I

A lei da reflexão é a mais antiga e foi enunciada por Herão de Alexandria no séc. II d.C. enquanto a

lei de Snel (grafia original). Refração nada mais é do que um fenômeno de espalhamento de fótons.

Metamateriais

Veja também: Walter S. Santos, Antonio Carlos F. Santos, Carlos E. Aguiar

American Brasil: Aula Aberta, n. 9, p. 58-

Tópicos de Eletromagnetismo I – Prof. Antônio Carlos

Leis da reflexão e refração

A lei da reflexão é a mais antiga e foi enunciada por Herão de Alexandria no séc. II d.C. enquanto a

(grafia original). Refração nada mais é do que um fenômeno de espalhamento de fótons.

Algumas concepções erradas: em alguns tratamentos de

polarização por reflexão, o ângulo de Brewster recebe toda a

atenção, deixando a impressão nada acontece em outros

de incidência. De fato, o grau de polarização da luz refletida varia

continuamente de 0 a 100 %.

Metamateriais (índice de refração negativo)

Walter S. Santos, Antonio Carlos F. Santos, Carlos E. Aguiar , Ótica com índice de refração negativo

-59, 2011

59

A lei da reflexão é a mais antiga e foi enunciada por Herão de Alexandria no séc. II d.C. enquanto a

(grafia original). Refração nada mais é do que um fenômeno de espalhamento de fótons.

Algumas concepções erradas: em alguns tratamentos de

polarização por reflexão, o ângulo de Brewster recebe toda a

atenção, deixando a impressão nada acontece em outros ângulos

de incidência. De fato, o grau de polarização da luz refletida varia

Ótica com índice de refração negativo , Scientific

Page 60: Topicos de Eletromagnetismo I

60 FIW 591 – Tópicos de Eletromagnetismo I – Prof. Antônio Carlos

Aula 24– Ondas Eletromagnéticas em condutores

Nome:______________________________________________________________________________

1- Em meio metálico de condutividade σ e permissividade elétrica ε é depositada

localmente uma densidade de carga ρo. Em quanto tempo a densidade cai a metade

de seu valor inicial?

A) ( ) ε/σ;

B) ( )( ε/σ)ln2;

C) ( ) ε/2σ;

D) ( ) 2ε/σ;

2- Para um dielétrico perfeito, a parte imaginária do número de onda vale:

A) ( )ω(εµ/2)1/2;

B) ( )ω(εµ)1/2;

C) ( )ω(2εµ)1/2;

D) ( ) 0;

3- Para um dielétrico perfeito, a parte imaginária do número de onda vale:

A) ( )ω(εµ/2)1/2;

B) ( )ω(εµ)1/2;

C) ( )ω(2εµ)1/2;

D) ( ) 0;

4- Três materiais possuem respectivamente condutividade elétricas σ1 =ωε, σ2 =200ωε,

σ1 =ωε/1000. É correto afirmar que:

A) ( ) o primeiro é condutor, o segundo é quase-condutor e o terceiro é isolante;

B) ( ) o primeiro é isolante, o segundo é condutor e o terceiro é quase-condutor;

C) ( ) o primeiro é quase-condutor, o segundo é isolante e o terceiro é condutor;

D) ( ) o primeiro é quase-condutor, o segundo é condutor e o terceiro é isolante;

5- Em eletrostática, vimos que um condutor blinda seu interior de campos elétricos

externos. Isto é verdade para:

A) ( ) ω> (Nq2/mεo)1/2;

B) ( ) ω< (Nq2/mεo)1/2;

C) ( ) ω= (Nq2/mεo)1/2;

D) ( ) qualquer frequência de onda;

6- É correto afirmar que a ionosfera:

A) ( ) é quase transparente para frequências baixas e absorve frequências altas;

B) ( ) é quase transparente para frequências altas e absorve frequências baixas;

C) ( ) é quase transparente para todas as frequências;

D) ( ) absorve igualmente em todas frequências;

Page 61: Topicos de Eletromagnetismo I

61 FIW 591 – Tópicos de Eletromagnetismo I – Prof. Antônio Carlos

Aula 24 – para casa

1- Aplicando o rotacional em t

BE

∂∂

−=×∇

rrr

e Et

EB

rr

rrµσµε +

∂∂

=×∇ obtenha as equações de

onda modificadas (equação de onda evanescente) para os campos elétrico e magnético em

metais (com ρ=0). Dica: AAArrr

2)()( ∇−⋅∇∇=×∇×∇ . (você não deverá gastar mais do

que poucas linhas)

2- Substituindo a solução ( )tkzi

oeEEω−=

rr na equação de onda modificada, mostre que k

2=µεω2

+iµσω é uma função complexa

Page 62: Topicos de Eletromagnetismo I

62 FIW 591 – Tópicos de Eletromagnetismo I – Prof. Antônio Carlos

Aula 25 – absorção e dispersão

Nome:______________________________________________________________________________

Objetivos: aprofundar nosso conhecimento sobre índice de refração.

1- A parte real de γωωω i

Em

q

xo

o

o −−

=22

é:

a) ( )

( )( ) 22222

22

ωγωω

ωω

+−

+

=o

oo

o

Em

q

x

b) ( )

( )( ) 22222

22

ωγωω

ωω

+−

=o

oo

o

Em

q

x

c) ( )

( )( ) 22222

22

ωγωω

ωω

−−

=o

oo

o

Em

q

x

d) ( )

( )( ) 22222

22

ωγωω

ωω

−−

+

=o

oo

o

Em

q

x

2- A dependência do índice de refração com a frequência da luz leva o nome de dispersão cromática, está mostrada na figura abaixo. Em geral, as transições atômicas mais intensas dos materiais transparentes

ocorrem na região do ultravioleta e assim, nas regiões do visível (0.4 a 0.7 µm) e infravermelho próximo

(0.7 a 2.5 µm) o índice de refração aumenta com a frequência (diminui com λ). Isto significa que quanto mais deslocado para o infravermelho for o comprimento de onda da luz,

a) ( ) menor será a sua velocidade de propagação; b) ( ) maior será a sua velocidade de propagação; c) ( ) maior será o índice refração; d) ( ) maior será a sua frequência;

Este fato é danoso na área de comunições óticas, pois o alargamento do sinal impõe um limite à taxa de repetição máxima possível de se transmitir por uma fibra ótica. S. C. Zilio, Optica Moderna – Fundamentos e Aplicações

3- Qual a cor da luz que é menos absorvida pelas plantas? A- ( ) vermelho; B-( ) amarelo;C-( ) verde;D-( ) violeta;

Page 63: Topicos de Eletromagnetismo I

63 FIW 591 – Tópicos de Eletromagnetismo I – Prof. Antônio Carlos

Aula 25 – absorção e dispersão (para casa)

1- Em 1871 Sellmeier deduziu a equação ∑ −+=

j j

jAn

2

0

2

2 1λλ

em que termos Aj são constantes; cada

λ0j é o comprimento de onda no vácuo associado à freqüência natural ν0j, tal que ν0jλ0j = c. Esta

formulação constitui uma melhoria considerável, do ponto de vista prático, sobre a equação de Cauchy.

Mostre que quando λ>> λ0j a equação de Cauchy é uma aproximação da equação de Sellmeier. Sugestão:

escreva a equação acima com o primeiro termo da soma; faça seu desenvolvimento com a expansão

binomial; tome a raiz quadrada de n2 e desenvolva-a novamente em série.

2- Para um oscilador harmônico amortecido ++dt

dx

mdt

xd γ2

2

Augustin Louis Cauchy (1787-1857)

construiu uma equação empírica para n(λ) para substâncias transparentes no visível. A sua

expressão é dada pela série de potências: n= C1 + C2λ-2

+C3λ-4

+... em que Ci são constantes. À

luz da figura abaixo, qual é o significado físico de C1?

Page 64: Topicos de Eletromagnetismo I

64 FIW 591 – Tópicos de Eletromagnetismo I – Prof. Antônio Carlos

Alguns fatos e curiosidades sobre absorção e dispersão

As viseiras dos capacetes dos astronautas são cobertas por uma fina de

ouro que reflete cerca de 70 % da luz incidente. Foi projetada para reduzir

a carga térmica no sistema de refrigeração, refletindo fortemente energia

radiante no infravermelho, mas transmitindo fortemente adequadamente

no visível. Existem no mercado óculos de sol baratos cobertos por

películas metálicas, e que funcionam do mesmo modo.

Na camada ionizada do topo da atmosfera os elétrons livres comportam-

se de um modo semelhante aos elétrons livres de um metal. O índice de

refração destas camadas é real e inferior a 1 para freqüências superiores a

freqüência de plasma. Para comunicar entre dois pontos geograficamente

distantes, é possível fazer refletir ondas de baixa freqüência na ionosfera.

Para se falar com alguém na Lua, é necessário utilizar sinais de alta

freqüência, para os quais a ionosfera é transparente.

Apesar da água ser essencialmente transparente, o vapor de água parece

esbranquiçado, tal como o vidro despolido. A razão é devido ao tamanho

do grão. Se for muito maior que os comprimentos de onda envolvidos, a

luz penetra em cada uma das partículas transparentes e é refletida e

refratada várias vezes antes de emergir. Não há qualquer distinção entre

qualquer das freqüências componentes e, portanto, a luz que chega ao

observador é branca (o mesmo vale para o açúcar, sal, nuvens, neve,

espuma da onda do mar, clara em neve, etc...). A tinta branca é

constituída por partículas transparentes (óxidos de zinco, titânio ou

chumbo) em suspensão num meio igualmente transparente. Quando os

índices de refração das partículas e do meio forem iguais, não há reflexão

e o conjunto fica transparente. Por outro lado, se os índices forem

diferentes, ocorrem reflexões para todos os comprimentos de onda e a

tinta aparece branca. Uma superfície difusora que absorva

uniformemente ao longo de todo o espectro, reflete um pouco menos que

uma superfície branca, parecendo acinzentada. Quanto menos refletir,

menos intenso será o cinzento; quando a absorção for quase total a

superfície parece preta.

Page 65: Topicos de Eletromagnetismo I

65 FIW 591 – Tópicos de Eletromagnetismo I – Prof. Antônio Carlos

Aula 26 – Guias de Ondas (o cabo coaxial)

Nome:______________________________________________________________________________

1- Um sinal harmônico atravessa um cabo coaxial com permeabilidade magnética relativa

µ/µo =1 e permissividade elétrica relativa ε/εo =2. A velocidade de fase deste sinal é

a) ( ) 2c;

b) ( ) c/2;

c) ( ) 21/2

c;

d) ( ) 2-1/2

c;

2- Tratando o vácuo como uma linha de transmissão, qual seria a indutância por unidade de

comprimento e a capacitância por unidade de comprimento do vácuo, respectivamente?

a) ( ) µo/2, εo/2

b) ( ) µo , εo

c) ( ) µo, 2εo

d) ( ) 2µo, εo

3- A impedância característica Zo de um cabo coaxial de indutância por unidade de

comprimento L e capacitância por unidade de comprimento C é: (dica: lembre-se que a

impedância de um meio é Z =(µ/ε)1/2

a) ( ) ( L/C )1/2

;

b) ( ) ( C/L )1/2

;

c) ( ) ( LC )1/2

;

d) ( ) ( LC )-1/2

;

4- Em um cabo coaxial de permissividade ε e permeabilidade µ, os campos elétrico e

magnético são dados por ρρ

ˆ)cos( wtkz

AE−

=r

e φρ

ˆ)cos(

c

wtkzAB

−=

r. Calcule o

vetor de Poynting.

Page 66: Topicos de Eletromagnetismo I

66 FIW 591 – Tópicos de Eletromagnetismo I – Prof. Antônio Carlos

Revisão para a terceira parte do curso:

Problemas do livro-texto: 9.1-9.4, 9.8-9.25.

Polarização

1- Considere duas vibrações com a mesma frequência e a mesma amplitude, porém com fases

diferentes, uma ao longo do eixo Ox, x=asen(ωt-α) e a outra ao longo do eixo Ou, y=asen(ωt-β).

As relações anteriores podem ser escritas na forma: (x/a)=sen(ωt)cos(α)-cos(ωt)sen(α)

(1),(y/a)=sen(ωt)cos(β)-cos(ωt)sen(β) (2). A) Multiplique a Equação (1)por senβ e a Equação (2)

por senα e a seguir subtraia as duas relações resultantes. B) Multiplique a Equação (1) por cosβ

e a Equação (2) por cosα e a seguir subtraia as duas relações resultantes. C) Eleve ao quadrado

e some os resultados dos itens (a) e (b). D) Deduza a equação x2+y

2 -2xycosδ=a

2sen

2δ, onde

δ=α-β. E) Use o resultado anterior para justificar cada um dos diagramas estudados em sala de

aula(polarização linear, circular e elíptica).

Energia e momento em ondas eletromagnéticas

1- O vetor de Poynting instantâneo é expresso por S=E××××H. O vetor de Poynting médio é obtido

integrando o vetor de Poynting instantâneo sobre um período e dividindo pelo período. Pode

também ser prontamente obtido em notação complexa ⟨S⟩=(1/2)Re(E××××H*). A) Calcule os

vetores de Poynting instantâneo e médio para E=(Eo eiωt

)j e H=(Ho ei(ωt-ε)

)k, onde j e k são os

unitários na direção y e z, respectivamente. B) Calcule a densidade de momento p=S/c2

instantâneo e médio.

Reflexão e transmissão

1- O ângulo crítico para reflexão interna total em uma interface que separa um líquido do ar é

igual a 42,5o. A) Sabendo que um raio de luz proveniente do líquido incide sobre a interface

com um ângulo de incidência de 35,0o, qual é o ângulo que o raio refratado no ar forma com a

normal? B) Sabendo que um raio de luz proveniente do ar incide sobre a interface com um

ângulo de incidência de 35,0o, qual é o ângulo que o raio refratado no líquido forma com a

normal?

2- Um feixe paralelo de luz não-polarizada proveniente do ar incide formando um ângulo de 54,5o

(com a normal) sobre uma superfície plana de vidro. O feixe refletido é completamente

linearmente polarizado. A) Qual é o índice de refração do vidro? B) Qual é o ângulo de refração

do feixe transmitido?

Dependência da permissividade com a frequência

1- Uma onda plana de 1 MHz de frequência (λ= 300 m) se propaga em um meio dispersivo e sem

perdas com velocidade de fase de 3×108 m/s. A velocidade de fase em função do comprimento

é dada pela relação v=kλ1/2, onde k é uma constante. Encontre a velocidade de grupo. Resp.

1,5×108 m/s.

2- A distância de um pulsar. Um pulsar é uma estrela de nêutrons muito densa que gira muito

rápido e que transmite pulsos de banda larga. Os pulsos são mais intensos na faixa entre 100 e

500 MHz. Os elétrons tornam o meio interestelar dispersivo permitindo medir a distancia do

pulsar. A velocidade de grupo da radiação de um pulsar é dada por vg =c[1-(fo/f)2]

1/2, onde

Page 67: Topicos de Eletromagnetismo I

FIW 591 – Tópicos de Eletromagnetismo I

c=3×108 m/s, f é a frequência do pulsar em Hz e f

é a frequência crítica de plasma em Hz. Para N=3

em Taylor (1+x)n ≈1+nx. A) mostre que a velocidade de grupo pode ser expressa como v

(1/2)(fo/f)2]. Como resultado da dispersão no meio interestelar, um pulso chega mais cedo a

altas frequências e mais tarde a frequências menores.B) Most

chegada ∆t de um pulso transmitido simultaneamente em duas frequências é dado por

∆t=(L/2c)[(fo/f2)2 –(fo/f1)

para f1 = 400 MHz e f2 = 300 MH

D) qual a diferença entre as velocidades dos pulsos. Resp: cerca de 1 parte em 100 bilhões,

mensurável somente pela grande distância.

3- Um arco-íris é produzido pela reflexão da luz solar em gotas de água

A Figura abaixo indica um raio que se refrata para o interior de uma gota no ponto A, é

refletido na superfície posterior da gota no ponto B e se refrata voltando para o ar no ponto C.

Os ângulos de incidência e refração,

incidência e de reflexão,

= θaA. B) Mostre que o ângulo em radianos antes de ele entrar na gota em A e depois que ele sai

da gota em C (a deflexão total do raio) é dado por

angular que ocorre em A, em B e em C e some para encontrar

escrever ∆ em termos de

forma quando o ângulo de deflexão

d∆/dθaA=0. Quando essa condição for obedecida, todos os raios próximos de

retornando na mesma direção, produzindo uma faixa brilhante no céu.

o qual isto ocorre. Mostre que cos

fórmula da derivada d(arc sen u(x))/dx=(1

a 1,342 para a luz violeta e 1,330 para a luz vermelha. Use os resultados dos itens (c) e (d) para

calcular θ1 e ∆ para a luz vermelha e para a luz violeta. Seus resultados da figura? Quando você

vê um arco-íris, qual das duas cores do arco

vermelha ou a violeta?

Tópicos de Eletromagnetismo I – Prof. Antônio Carlos

m/s, f é a frequência do pulsar em Hz e fo=9N1/2

. N é a densidade de elétrons (m

é a frequência crítica de plasma em Hz. Para N=3×104 m

-3 , fo=1,56 kHz << f. Assim,

1+nx. A) mostre que a velocidade de grupo pode ser expressa como v

]. Como resultado da dispersão no meio interestelar, um pulso chega mais cedo a

altas frequências e mais tarde a frequências menores.B) Mostre que diferença no tempo de

t de um pulso transmitido simultaneamente em duas frequências é dado por

)2], onde L é a distância do pulsar. C)Se a diferença de tempo é

= 300 MHz, com N=3×104 m

-3 (30 e

-/litro), calcule L. Resp. 6060 anos

D) qual a diferença entre as velocidades dos pulsos. Resp: cerca de 1 parte em 100 bilhões,

mensurável somente pela grande distância.

íris é produzido pela reflexão da luz solar em gotas de água esféricas existentes no ar.

A Figura abaixo indica um raio que se refrata para o interior de uma gota no ponto A, é

refletido na superfície posterior da gota no ponto B e se refrata voltando para o ar no ponto C.

Os ângulos de incidência e refração, θa e θb, são indicados nos pontos A e C, e os ângulos

incidência e de reflexão, θa e θr, são indicados no ponto B. A)Mostre que θaB

. B) Mostre que o ângulo em radianos antes de ele entrar na gota em A e depois que ele sai

C (a deflexão total do raio) é dado por ∆=2θaA

-4θbA +π. (Dica: Determine a deflexão

angular que ocorre em A, em B e em C e some para encontrar ∆.) C) Use a lei de Snell para

em termos de θaA e de n, o índice de refração da água na gota. D) O ar

forma quando o ângulo de deflexão ∆ é estacionário em relação a θaA

=0. Quando essa condição for obedecida, todos os raios próximos de

retornando na mesma direção, produzindo uma faixa brilhante no céu. Chame

o qual isto ocorre. Mostre que cos2θ1 =(1/3)(n

2-1). (Dica: Talvez você ache conveniente usar a

fórmula da derivada d(arc sen u(x))/dx=(1-u2)

-1/2 (du/dx)). E) O índice de refração da água é igual

a 1,342 para a luz violeta e 1,330 para a luz vermelha. Use os resultados dos itens (c) e (d) para

para a luz vermelha e para a luz violeta. Seus resultados da figura? Quando você

is, qual das duas cores do arco-íris primário está mais afastada do horizonte, a

67

. N é a densidade de elétrons (m-3

) e fo

=1,56 kHz << f. Assim, expandindo

1+nx. A) mostre que a velocidade de grupo pode ser expressa como vg ≈c[1-

]. Como resultado da dispersão no meio interestelar, um pulso chega mais cedo a

re que diferença no tempo de

t de um pulso transmitido simultaneamente em duas frequências é dado por

], onde L é a distância do pulsar. C)Se a diferença de tempo é ∆t=1,13 s

/litro), calcule L. Resp. 6060 anos-luz.

D) qual a diferença entre as velocidades dos pulsos. Resp: cerca de 1 parte em 100 bilhões,

esféricas existentes no ar.

A Figura abaixo indica um raio que se refrata para o interior de uma gota no ponto A, é

refletido na superfície posterior da gota no ponto B e se refrata voltando para o ar no ponto C.

, são indicados nos pontos A e C, e os ângulos B

= θbA, θa

C = θb

A e θb

C

. B) Mostre que o ângulo em radianos antes de ele entrar na gota em A e depois que ele sai

. (Dica: Determine a deflexão

.) C) Use a lei de Snell para

e de n, o índice de refração da água na gota. D) O arco-íris se A, ou seja, quando

=0. Quando essa condição for obedecida, todos os raios próximos de θaA sairão da gota

Chame θ1 o ângulo para

1). (Dica: Talvez você ache conveniente usar a

/dx)). E) O índice de refração da água é igual

a 1,342 para a luz violeta e 1,330 para a luz vermelha. Use os resultados dos itens (c) e (d) para

para a luz vermelha e para a luz violeta. Seus resultados da figura? Quando você

íris primário está mais afastada do horizonte, a

Page 68: Topicos de Eletromagnetismo I

FIW 591 – Tópicos de Eletromagnetismo I

Primeira Prova de

Nome:_______________________________________________________________________

Caro aluno, você deve escolher

1-(2,5) (a) Considere que pode ser escrito como o gradiente de um escalar? Encontre um potencial escalar que funcione. Qual pode ser escrito como o rotacional de um vetor? Encontre um potencial vetorial adequado.

2- (2,5) Verifique o teorema de Stokes para a função superfície triangular mostrada na figura ao lado

3-(2,5) Uma região esférica tem carga uniforme por unidade de

volume ρ. Seja r o vetor do centro da esfera até um ponto genérico P dentro desta. (a) Calcule o campo elétrico em P. (b) Uma cavidadeesférica é criada na esfera, conforme indicado na figura ao lado (vetor que liga o centro da esfera com o centro da cavidade). Usaconceito de superposição, calcule o campo elétrico em todos os pontos no interior da cavidade.

4-(2,5) A carga por unidade de comprimento uniformemente ao longo de um bastão delgado de comprimento L. (a) Determine o potencial (escdistância y de uma das pontas do bastão em lao lado). (b) Utilize o resultado de (a) para calcular a componente do campo elétrico em P na direção y (ao longo do bastão).(c) Determine a componente do campo elétrico em P na direção perpendicular ao bastão.

5-(2,5) Um cilindro condutor longo tem raio R. O Sejam A, B e C pontos situados, respectivamente, a R/2, R, 3R de distância do eixo do cilindro. Determine (a) o campo elétrico potencial VA – VC.

Tópicos de Eletromagnetismo I – Prof. Antônio Carlos

Universidade Federal do Rio de Janeiro Instituto de Física

Curso de Licenciatura em Física Primeira Prova de Tópicos de Eletromagnetismo I (2012/1)

Prof. Antônio Carlos ([email protected])

Nome:_______________________________________________________________________

Caro aluno, você deve escolher quatro questões para resolver. Boa sorte!

(2,5) (a) Considere que F1 = x2 k e F2 = xi+yj+zk. Calcule a divergência e o rotacional de

pode ser escrito como o gradiente de um escalar? Encontre um potencial escalar que funcione. Qual pode ser escrito como o rotacional de um vetor? Encontre um potencial vetorial adequado.

(2,5) Verifique o teorema de Stokes para a função v=yz, usando a superfície triangular mostrada na figura ao lado.

(2,5) Uma região esférica tem carga uniforme por unidade de

o vetor do centro da esfera até um ponto genérico P dentro desta. (a) Calcule o campo elétrico em P. (b) Uma cavidade esférica é criada na esfera, conforme indicado na figura ao lado (a é o vetor que liga o centro da esfera com o centro da cavidade). Usando o conceito de superposição, calcule o campo elétrico em todos os pontos no interior da cavidade.

A carga por unidade de comprimento λ é distribuída uniformemente ao longo de um bastão delgado de comprimento L. (a) Determine o potencial (escolhendo zero no infinito) no ponto P a uma distância y de uma das pontas do bastão em linha com ele (vide figura

). (b) Utilize o resultado de (a) para calcular a componente do campo elétrico em P na direção y (ao longo do bastão).(c) Determine a

mponente do campo elétrico em P na direção perpendicular ao

(2,5) Um cilindro condutor longo tem raio R. O módulo do campo elétrico na superfície do cilindro é E

Sejam A, B e C pontos situados, respectivamente, a R/2, R, 3R de distância do eixo do cilindro. Determine (a) o campo elétrico (vetor) nos pontos A, B e C. b) A diferença de potencial V

68

(2012/1)

Nome:_______________________________________________________________________

questões para resolver. Boa sorte!

e o rotacional de F1 e F2 . Qual deles pode ser escrito como o gradiente de um escalar? Encontre um potencial escalar que funcione. Qual pode ser escrito como o rotacional de um vetor? Encontre um potencial vetorial adequado.

campo elétrico na superfície do cilindro é Eo. Sejam A, B e C pontos situados, respectivamente, a R/2, R, 3R de distância do eixo do cilindro. Determine

nos pontos A, B e C. b) A diferença de potencial VB-VC. c) A diferença de

Page 69: Topicos de Eletromagnetismo I

69 FIW 591 – Tópicos de Eletromagnetismo I – Prof. Antônio Carlos

Gabarito da primeira prova de Tópicos de Eletromagnetismo I (2012/1)

( )

( )

0

ˆˆ

ˆ2

00

ˆˆ

3111ˆˆˆˆ.

0ˆˆˆ.

2

2

1

2

2

1

r

)

rr

)

rr

))rr

)rr

=∂∂

∂∂

∂∂

=×∇

−=∂∂

∂∂

∂∂

=×∇

=++=++⋅

∂∂

+∂∂

+∂∂

=∇

=⋅

∂∂

+∂∂

+∂∂

=∇

zyx

zyx

zyx

F

yx

x

zyx

zyx

F

zzyyxxzz

yy

xx

F

zxzz

yy

xx

F

Como o rot(gradϕ)=0 e rot(F2)=0, F2 pode ser escrito como gradϕ, onde ϕ é uma função

escalar.

Como div(rotA)=0, para qualquer A e div(F1)=0, então F1 pode ser escrito como o rotacional

de A.

( )

222

),(2

),(2

),(2

ˆˆˆˆ

222

2

2

2

2

zyx

xyfz

zz

zxfy

yy

zyfx

xx

zz

yy

xx

zzyyxxF

++=

+=

=∂∂

+=

=∂∂

+=

=∂∂

∂∂

+∂∂

+∂∂

=++=

ϕ

ϕ

ϕ

ϕ

ϕ

ϕ

ϕ

ϕ))r

Page 70: Topicos de Eletromagnetismo I

70 FIW 591 – Tópicos de Eletromagnetismo I – Prof. Antônio Carlos

yx

A

xA

xx

A

xy

A

x

A

x

A

z

A

z

A

y

A

zx

AAA

zyx

zyx

AF

y

y

xy

zx

yz

zyx

ˆ3

3

0

0

ˆ

ˆˆ

3

3

2

2

2

1

=

=

=∂

=

∂∂

−∂

=

∂∂

−∂∂

=

∂−

∂∂

=∂∂

∂∂

∂∂

=×∇=

r

)

rrr

2)

( )

zdzydyxdxld

ldzyldzyldzyldv

ldvSdv

C

S C

ˆˆˆ

).ˆ().ˆ().ˆ( 321

++=

++=⋅

⋅=×∇

∫ ∫∫∫

∫∫ ∫

r

rrrrr

rrrrr

x

yzyx

zyx

v ˆ

00

ˆˆ

=∂∂

∂∂

∂∂

=×∇

)

rr

Como o rot v aponta na direção x, pela regra da mão direita, a integral de caminho

será percorrida no sentido anti-horário. (a,0,0)→(0,2a,0)→(0,0,a)→(a, 0,0)

(a,0,0)→(0,2a,0)

y = 2a- 2x → dy=-2dx, dz=0

ydxxdxld

ldzy

ˆˆ

0).ˆ(

1

1

−=

=∫r

r

(0,2a,0)→(0,0,a)

z=a-y/2 →dz=-dy/2, dx = 0

Page 71: Topicos de Eletromagnetismo I

71 FIW 591 – Tópicos de Eletromagnetismo I – Prof. Antônio Carlos

( )

zdy

ydyld

aayydy

zdy

ydyzyldzya a

ˆ2

ˆ

404

1

42)ˆ

2ˆ).(ˆ().ˆ(

2

0

2

22

0

2

2

2

−=

=−−=−=−=−= ∫∫∫r

r

(0,0,a)→(a,0,0)

z=a-x →dz=-dx, dy = 0 e y=0

zdzxdxld

zdzxdxzyldzy

ˆˆ

0)ˆˆ).(ˆ().ˆ(

2

2

−=

=−= ∫∫r

r

Então:

2aldvC

=⋅∫rr

Agora, calculando o fluxo do rotacional. Neste caso é ainda mais simples, pois

como envolve o produto escalar do rotacional de v que é constante e que aponta

na direção x com o elemento de área dS, apenas a componente x de dS entra no

calculo. A área perpendicular à direção x é a projeção no plano yz , ou seja

2a×a/2= a2. Vamos verificar?

( ) ( )

2/

22)22(

)ˆˆˆ(ˆ

0

222

0

222

0

0

22

00

yaz

aaazazdzzaydz

dydzdydzydxdzzdxdyxdydzxSdv

aaza

a

zaa

sS S

−=

=−=−=−==

==++⋅=⋅×∇

∫∫

∫∫∫∫∫∫ ∫∫

−rrr

3)

A) pela Lei de Gauss

rr

E

rE

rrE

dVSdE

o

o

o

S V

o

ˆ3

3

3

44

.

32

ερ

ερ

πρπε

ρε

=

=

=

=∫∫ ∫∫∫

r

rr

Page 72: Topicos de Eletromagnetismo I

FIW 591 – Tópicos de Eletromagnetismo I

B) Pelo principio da superposição, o campo em um ponto dentro da cavidade pode ser

escrito como o campo da esfera maciça (item a) subtraído do campo devido à cavidade

preenchida com densidade de carga rô, ou

Considerando

'ˆ3

''

3

''

'4

.

2

rr

E

rE

rE

SdE

o

o

o

S

o

ερ

ερ

ρπε

ε

=

=

=

=∫∫

r

rr

Onde r’ está centrado em

Então, pelo principio da superposição:

E

EE

cavidade

esferacavidade

r

rr

ερ3

=

=

onde Eesfera é o campo calculado no item a).

onde, pela figura abaixo:

4)

a) = ∫PVo4

1)(

πε

b)

Tópicos de Eletromagnetismo I – Prof. Antônio Carlos

Pelo principio da superposição, o campo em um ponto dentro da cavidade pode ser

escrito como o campo da esfera maciça (item a) subtraído do campo devido à cavidade

preenchida com densidade de carga rô, ou seja:

a cavidade com densidade ρ:

'

3

'4 3r

dVV

πρ

ρ∫∫∫

Onde r’ está centrado em a

Então, pelo principio da superposição:

arrrrrr

rr

E

oooo

esfera

r

r

ερ

ερ

ερ

ερ

3)'ˆ'ˆ(

3'ˆ

3

'

=−=−

−,

é o campo calculado no item a).

onde, pela figura abaixo: r’ + a = r.

===

++

∫ ∫ rr

dr

r

dq

o

Ly

yo

Ly

yo

ln4

ln44

1

πελ

πελλ

πε

72

Pelo principio da superposição, o campo em um ponto dentro da cavidade pode ser

escrito como o campo da esfera maciça (item a) subtraído do campo devido à cavidade

+y

Ly

Page 73: Topicos de Eletromagnetismo I

73 FIW 591 – Tópicos de Eletromagnetismo I – Prof. Antônio Carlos

yLyy

yyLy

E

yy

VVE

oo

ˆ11

11

4

ˆ

+−=

+−=

∂−=∇−=

πελ

πελr

rr

Ou ainda:

yLyy

yrr

ydr

r

rdqE

o

Ly

yo

Ly

yoo

ˆ11

1

4

ˆ

4

4

122

+−=

−===++

∫∫ πελ

πελλ

πεπε

r,

onde y chapéu aponta verticalmente para cima.

c) Pela simetria, a componente perpendicular do campo elétrico é nula.

5) Um cilindro condutor longo tem raio R. O módulo do campo elétrico na superfície do cilindro é Eo.

Sejam A, B e C pontos situados, respectivamente, a R/2, R, 3R de distância do eixo do cilindro.

Determine (a) o campo elétrico (vetor) nos pontos A, B e C. b) A diferença de potencial VB-VC. c) A

diferença de potencial VA – VC.

a) Como o cilindro é condutor, a carga situa-se totalmente na superfície. Então, tomando

como uma gaussiana uma superfície cilíndrica infinita de raio R/2, a carga no interior desta

superfície (ponto A) é nula, então ∫∫ ==S

oqSdE 0/. εrr

. Logo E = 0 no ponto A.

No ponto B, o módulo do campo é Eo e como o campo deve ser perpendicular à superfície,

ρoEE =r

, onde rô chapéu é o unitário na direção radial.

No ponto C, aplicamos a Lei de Gauss: ∫∫ =S

oqSdE ε/.rr

, onde q é a carga na superfície

do cilindro. Escolhendo uma gaussiana de raio 3R (figura),

o

o

S

o

rl

qE

qrlE

qSdE

επ

επ

ε

2

/2

/.

=

=

=∫∫rr

Para qualquer r>R

Onde l é o comprimento do cilindro.

E pela simetria, ρEE =r

. Temos agora que encontrar a carga q. Como na superfície o

campo tem módulo Eo, então Eo=q/2πRlεo (I),

Page 74: Topicos de Eletromagnetismo I

FIW 591 – Tópicos de Eletromagnetismo I

onde fizemos r=R. Então, substituindo (I) no resultado anterior, temos que fora do

cilindro: E=EoR/r. Como r = 3R:

d) Como o cilindro é condutor o potencial na superfície (ponto B) é o mesmo no ponto C.

Então VB-Vc=0.

e) De novo VA –Vc = V

3ln

.3

R

RRE

dEVV

o

R

R

BA

−=

−=− ∫r

Tópicos de Eletromagnetismo I – Prof. Antônio Carlos

onde fizemos r=R. Então, substituindo (I) no resultado anterior, temos que fora do

R/r. Como r = 3R: 3

ˆ

3

ˆ ρρ oo E

R

REE ==r

(no ponto C)

Como o cilindro é condutor o potencial na superfície (ponto B) é o mesmo no ponto C.

= VA – VB .

( )3ln

ln)ˆ.(ˆ

33

RE

REr

dr

REdrr

REld

o

R

R

o

o

R

R

o

=

−=−=

−= ∫∫ ρρr

74

onde fizemos r=R. Então, substituindo (I) no resultado anterior, temos que fora do

Como o cilindro é condutor o potencial na superfície (ponto B) é o mesmo no ponto C.

ln3

rR

R=

Page 75: Topicos de Eletromagnetismo I

FIW 591 – Tópicos de Eletromagnetismo I

Segunda Prova de

Caro aluno, você deve escolher

4-(2,5) Uma espira circular de fio de diâmetro d é posicionada de tal forma que a normal faz um ângulo

θ com a direção de um campo magnético uniforme B

normal gira em um cone em volta da direção do campo

normal e o sentido do campo (θ) perman

5-(2,5) Escreva as equações de Maxwell e as suas equações constitutivas, explique

detalhadamente o significado f

ilustrar. Por quê as equações de Maxwell não são totalmente simétricas?

Tópicos de Eletromagnetismo I – Prof. Antônio Carlos

Universidade Federal do Rio de Janeiro Instituto de Física

Curso de Licenciatura em Física Segunda Prova de Tópicos de Eletromagnetismo I (2012/1)

Prof. Antônio Carlos ([email protected])

Caro aluno, você deve escolher quatro questões para resolver. Boa sorte!

1 – (2,5) A figura ao lado mostra uma seção reta de uma fita longa e

fina de largura w que está conduzindo uma corrente

uniformemente distribuída i para dentro do papel.

densidade linear de corrente κκκκ=dI/dL. B)(2,0)

vetores unitários, qual é o campo B em um ponto P no plano da fita

situado a uma distância d de uma das bordas? (sugestão: imagine a

lâmina sendo construída de muitos fios, longos e paralelos

fácil) ou utilize ∫∫×

=S

dSr

rB '

ˆ

4 2

κπµ

rr

-mais trabalhoso

2- (2,5) A figura ao lado mostra uma seção reta de um condutor

cilíndrico oco de raios a e b que conduz uma corrente

uniformemente distribuída para dentro do papel

vetores unitários apropriados: a) Calcule a densidade de corrente

volumar j; b) Calcule o campo magnético (vetor)

ou seja, r < b; b < r < a; r> a. c) Esboce o campo em função de r;

3-(2,5) A figura ao lado mostras as placas de um capacitor de placas

paralelas de raio R separados por uma distância d. Elas est

conectadas, conforme mostrado, a longos fios nos quais existe uma

corrente condução constante i. Também estão mostrados três

círculos hipotéticos de raio r, dois deles fora do capacitor e um

entre as placas. A)(1,5) Calcule o campo magnético na

circunferência de cada um destes círculos; B) (0,5) Calcule o campo

elétrico (vetor) entre as placas do capacitor;

corrente de deslocamento entre as placas do capacitor;

(2,5) Uma espira circular de fio de diâmetro d é posicionada de tal forma que a normal faz um ângulo

com a direção de um campo magnético uniforme Bo. A espira está “oscilando” de tal forma que a sua

normal gira em um cone em volta da direção do campo à uma taxa constante ω

) permanece imutável durante o processo. Que f.e.m. surge na espira?

(2,5) Escreva as equações de Maxwell e as suas equações constitutivas, explique

detalhadamente o significado físico de cada uma das equações; utilize desenhos para

Por quê as equações de Maxwell não são totalmente simétricas?

75

(2012/1)

questões para resolver. Boa sorte!

(2,5) A figura ao lado mostra uma seção reta de uma fita longa e

fina de largura w que está conduzindo uma corrente

uniformemente distribuída i para dentro do papel. A) (0,5) Calcule a

B)(2,0) Em termos dos

em um ponto P no plano da fita

(sugestão: imagine a

lâmina sendo construída de muitos fios, longos e paralelos (mais

mais trabalhoso).

(2,5) A figura ao lado mostra uma seção reta de um condutor

que conduz uma corrente i

para dentro do papel. Em termos dos

a) Calcule a densidade de corrente

ético (vetor) em todo o espaço,

c) Esboce o campo em função de r;

(2,5) A figura ao lado mostras as placas de um capacitor de placas

paralelas de raio R separados por uma distância d. Elas estão

conectadas, conforme mostrado, a longos fios nos quais existe uma

. Também estão mostrados três

círculos hipotéticos de raio r, dois deles fora do capacitor e um

Calcule o campo magnético na

; B) (0,5) Calcule o campo

entre as placas do capacitor; C) (0,5) calcule a

corrente de deslocamento entre as placas do capacitor;

(2,5) Uma espira circular de fio de diâmetro d é posicionada de tal forma que a normal faz um ângulo

. A espira está “oscilando” de tal forma que a sua

ω; o ângulo entre a

ce imutável durante o processo. Que f.e.m. surge na espira?

(2,5) Escreva as equações de Maxwell e as suas equações constitutivas, explique

ísico de cada uma das equações; utilize desenhos para

Por quê as equações de Maxwell não são totalmente simétricas?

Page 76: Topicos de Eletromagnetismo I

76 FIW 591 – Tópicos de Eletromagnetismo I – Prof. Antônio Carlos

Gabarito

1- A) κκκκ=dI/dL= -(i/w)j

B)

κκκκ=-(i/w)k

Primeiro modo (mais trabalhoso):

( )( ) ( )

( ) ( )

[ ] jd

wd

w

i

x

dxj

w

i

x

dxj

w

iB

yxx

ydy

yx

dyyx

xdxjw

ijdx

yx

wix

dyB

jyxw

ixr

yxr

yx

jyix

r

rr

jyixr

kw

i

dSr

rB

o

wd

d

wd

d

wd

d

wd

d

S

ˆln2

ˆ2

)1(1ˆ4

1

1ˆ4

ˆ4

ˆˆ

ˆˆˆ

ˆˆ

ˆ

4

2/12222/322

2/3222/322

22

222

22

2

+==−−=

+=

+

+=

+=

+=×

+=

+

−−==

−−=

−=

×=

∫∫

∫ ∫∫ ∫

∫∫

+∞+

∞−

+

+ ∞+

∞−

∞+

∞−

+

πµ

πµ

πµ

πµ

πµ

κ

κ

κπµ

r

r

r

r

r

r

rr

Segundo modo (bem mais fácil): Considerando a superfície constituda por fios (B=µoi/2πr),

então dB=µodi/2πrj (pela regra da mão direita) com r=x e di=(i/w)dx

jw

wd

w

ix

w

ij

x

dx

w

iB owd

d

o

wd

d

o ˆln2

ln2

ˆ2

+===

++

∫ πµ

πµ

πµr

2- A) j=i/área; j=-i/π(a2 – b

2)k

B)

Page 77: Topicos de Eletromagnetismo I

77 FIW 591 – Tópicos de Eletromagnetismo I – Prof. Antônio Carlos

( )

( )

φπ

µ

µπ

µ

φπ

µ

ππ

µπ

ππ

µµ

µ

ˆ2

2

.

ˆ)(2

2

2..

0

0

.

22

22

2

22

22

r

iB

irB

ildB

ar

r

br

ba

iB

rba

irB

rdrba

iSdjldB

arb

B

ibr

ildB

o

o

o

o

r

b

o

r

b

oo

o

−=

=

=

>

−−

−=

−=

−==

<<

=

=→<

=

∫ ∫∫∫

r

rr

r

rrrr

rr

rr

C)fazer o gráfico

3-a) nos dois fios

φπ

µ

µπ

µ

ˆ2

2

.

r

iB

irB

ildB

o

o

o

=

=

=∫

r

rr

Entre as placas do capacitor

Page 78: Topicos de Eletromagnetismo I

78 FIW 591 – Tópicos de Eletromagnetismo I – Prof. Antônio Carlos

φπ

µ

µππ

µεπ

εµπ

επ

επεπεσ

σε

σε

εµ

ˆ2

2

)(

)1(2

.

..

2

22

2

22

r

iB

iRR

idS

R

irB

R

i

dt

dE

totalR

it

R

QE

placaSES

dSSdE

mas

SdEdt

dldB

o

oo

So

oo

o

ooo

o

S

o

C S

oo

=

===

=

===

=

=

=

∫∫

∫∫ ∫

∫ ∫∫

r

rr

rrrr

b) Como calculado no item anterior:

)(

)1(2

.

22total

R

it

R

QE

placaSES

dSSdE

ooo

o

S

o

επεπεσ

σε

σε

===

=

=∫∫ ∫rr

c)

22 R

i

R

it

tt

E

t

Dj

o

ood πεπεε =

∂∂

=∂∂

=∂∂

=

4-como o ângulo não muda, a f.e.m é nula

5- Vide livro;

Page 79: Topicos de Eletromagnetismo I

FIW 591 – Tópicos de Eletromagnetismo I

Terceira Prova de

Caro aluno, você deve escolher

1- (2,5) A componente elétrica de um feixe de luz polarizada

V/m)sen(1,00×106 z+ωt)

onda, incluindo o valor númerico de

poyinting; (d) a intensidade da luz; (e) a densidade de energia.

2- (2,5) (modelo de Lorentz) A equação de um oscilador forçado com amortecimento é: 2xmxmxm oeee ++ ωγ&&&

termo; B) Seja E=Eoeiω

expressão acima e obtenha

a variação de α com ω, para

3- (2,5) Uma onda eletromagnética cuja parte elétrica possui amplitude E

um meio de impedância Z

de um meio 2 com impedância Z

Z2>>Z1.

4- (2,5) Descreva completamente

onde B representa o campo magnético. Dica: lembre

campo elétrico.

5- (2,5) Encontre as velocidades de fase e de grupo para uma onda de 100 MH

dispersivo sem perdas no qual a velocidade de fase é v

Tópicos de Eletromagnetismo I – Prof. Antônio Carlos

Universidade Federal do Rio de Janeiro Instituto de Física

Curso de Licenciatura em Física Terceira Prova de Tópicos de Eletromagnetismo I (2012/1)

Prof. Antônio Carlos ([email protected])

Caro aluno, você deve escolher quatro questões para resolver. Boa sorte!

(2,5) A componente elétrica de um feixe de luz polarizada no vácuo é dada por

t)j . a) Escreva uma expressão para a componente magnética

onda, incluindo o valor númerico de ω. Determine (b) o comprimento de onda; (c) o

intensidade da luz; (e) a densidade de energia.

(2,5) (modelo de Lorentz) A equação de um oscilador forçado com amortecimento é:

)(teEx = , onde e é a carga do elétron. A) Explique o significado de cada

ωt e x=xoe

i(ωt-α), em que Eo e xo são grandezas reais. Substitu

expressão acima e obtenha xo; C) Deduza uma expressão para a diferença de fase,

, para ω<<ωo, ω=ωo e ω>>ωo.

Uma onda eletromagnética cuja parte elétrica possui amplitude Eo e que se propaga em

um meio de impedância Z1 incide normalmente (θi=0o) sobre a superfície que separa o meio 1

de um meio 2 com impedância Z2. Calcule as amplitudes das ondas transmitida e refletida para

Descreva completamente o estado de polarização da onda: B=Bo[cos(kz

onde B representa o campo magnético. Dica: lembre-se que a polarização é definida através do

Encontre as velocidades de fase e de grupo para uma onda de 100 MH

dispersivo sem perdas no qual a velocidade de fase é vf = 2×107λ2/3

(m/s).

79

(2012/1)

questões para resolver. Boa sorte!

é dada por E =(5,00

. a) Escreva uma expressão para a componente magnética (vetor)

primento de onda; (c) o vetor de

(2,5) (modelo de Lorentz) A equação de um oscilador forçado com amortecimento é:

é a carga do elétron. A) Explique o significado de cada

são grandezas reais. Substitua-as na

; C) Deduza uma expressão para a diferença de fase, α, e discuta

e que se propaga em

) sobre a superfície que separa o meio 1

Calcule as amplitudes das ondas transmitida e refletida para

[cos(kz-ωt)i+sen(kz-ωt)j],

se que a polarização é definida através do

Encontre as velocidades de fase e de grupo para uma onda de 100 MHz em um meio

Page 80: Topicos de Eletromagnetismo I

FIW 591 – Tópicos de Eletromagnetismo I

Caro aluno, você deve escolher

1 – (2,5) Uma lâmina plana infinita não

no plano y=-0,6 m. Uma segunda lâmina plana infi

está no plano x=1 m. Finalmente, uma casca esférica não

na interseção dos dois planos carregados, tem uma

magnitude, a direção e o sentido do campo elétrico no eixo x

3

cilíndrico de raio R contendo um furo longo cilíndrico de raio a. Os eixos dos dois

cilindros são paralelos e estão separados

está uniformemente distribuída ao longo da área sombreada na figura. (a) use

conceitos de superposição para calcular o campo magnético no centro do furo.

(b) Discuta os dois casos especiais a=0 e b =0. (c) É possível usar

para mostrar que o campo magnético no furo é uniforme? (sugestão: considere

o furo cilíndrico preenchido com duas correntes iguais movendo

opostos, cancelando

tenha a mes

campos devidos aos dois cilindros completos de corrente, de raios R e a, cada

cilindro com a mesma densidade de corrente.)

4-(2,5) Considere duas cascas esféricas metálicas,

esférica tem uma carga Q, mas a interna está aterrada. Isso significa que o potencial na casca interna é

o mesmo que o potencial nos pontos afastados das cascas. Determine a carga na casca interna.

5-(2,5) Deduza a equação de onda para o campo magnético

Tópicos de Eletromagnetismo I – Prof. Antônio Carlos

Universidade Federal do Rio de Janeiro Instituto de Física

Curso de Licenciatura em Física Prova de Final de Tópicos de Eletromagnetismo I (2012/2)

Prof. Antônio Carlos ([email protected])

Caro aluno, você deve escolher quatro questões para resolver. Boa sorte!

Uma lâmina plana infinita não-condutora tem uma densidade superficial de carga

. Uma segunda lâmina plana infinita não condutora tem densidade superficial

. Finalmente, uma casca esférica não-condutora com raio R=1m

na interseção dos dois planos carregados, tem uma densidade superficial de carga σmagnitude, a direção e o sentido do campo elétrico no eixo x em x=0,4 m e x=2,5 m.

2-(2,5) Um fio de massa M e comprimento L está suspenso

por um par de contatos flexíveis na presença de um campo

magnético uniforme de módulo B apontando para dentro

do papel. Determine (a) o valor absoluto e (b) o sentido da

corrente necessária para remover a tensão dos contatos.

3-(2,5) A figura ao lado mostra a seção transversal de um condutor longo

cilíndrico de raio R contendo um furo longo cilíndrico de raio a. Os eixos dos dois

cilindros são paralelos e estão separados por um distância b. Uma corrente i

está uniformemente distribuída ao longo da área sombreada na figura. (a) use

conceitos de superposição para calcular o campo magnético no centro do furo.

(b) Discuta os dois casos especiais a=0 e b =0. (c) É possível usar

para mostrar que o campo magnético no furo é uniforme? (sugestão: considere

o furo cilíndrico preenchido com duas correntes iguais movendo

opostos, cancelando-se uma à outra. Suponha que cada uma dessas correntes

tenha a mesma densidade de corrente do condutor real. Assim, superponha os

campos devidos aos dois cilindros completos de corrente, de raios R e a, cada

cilindro com a mesma densidade de corrente.)

as cascas esféricas metálicas, finas e concêntricas, de raio a e b, onde b>a. A casca

esférica tem uma carga Q, mas a interna está aterrada. Isso significa que o potencial na casca interna é

o mesmo que o potencial nos pontos afastados das cascas. Determine a carga na casca interna.

) Deduza a equação de onda para o campo magnético

80

(2012/2)

questões para resolver. Boa sorte!

condutora tem uma densidade superficial de carga σ1=+3µC/m2 e está

tem densidade superficial σ2 = -2µC/m2 e

=1m e com centro no plano z=0

σ3 = -3µC/m2 . Determine a

(2,5) Um fio de massa M e comprimento L está suspenso

por um par de contatos flexíveis na presença de um campo

magnético uniforme de módulo B apontando para dentro

do papel. Determine (a) o valor absoluto e (b) o sentido da

rente necessária para remover a tensão dos contatos.

(2,5) A figura ao lado mostra a seção transversal de um condutor longo

cilíndrico de raio R contendo um furo longo cilíndrico de raio a. Os eixos dos dois

por um distância b. Uma corrente i

está uniformemente distribuída ao longo da área sombreada na figura. (a) use

conceitos de superposição para calcular o campo magnético no centro do furo.

(b) Discuta os dois casos especiais a=0 e b =0. (c) É possível usar a lei de Ampère

para mostrar que o campo magnético no furo é uniforme? (sugestão: considere

o furo cilíndrico preenchido com duas correntes iguais movendo-se em sentidos

se uma à outra. Suponha que cada uma dessas correntes

ma densidade de corrente do condutor real. Assim, superponha os

campos devidos aos dois cilindros completos de corrente, de raios R e a, cada

e concêntricas, de raio a e b, onde b>a. A casca

esférica tem uma carga Q, mas a interna está aterrada. Isso significa que o potencial na casca interna é

o mesmo que o potencial nos pontos afastados das cascas. Determine a carga na casca interna.

Page 81: Topicos de Eletromagnetismo I

FIW 591 – Tópicos de Eletromagnetismo I

Segunda Chamada de

Caro aluno, você deve escolher

1- (2,5) A densidade volumar de uma distribuição de cargas é, em coordenadas esféricas

r<b)=ρo, ρ (b≤ r<2a)=-ρo, ρ (r≥a)=0, onde a, b e

a) Esboce um gráfico da densidade contra r;

b) Determine o raio b que anula a carga total da distribuição;

c) Dê o vetor deslocamento

item anterior;

d) Faça um gráfico da componente radial de

2-(2,5) Uma corrente constante, retilínea e ilimitada I flui ao longo do eixo z de um sistema de coordenadas

cilíndricas, no seu sentido positivo. Calcule explicitamente a integral curvilínea (de caminho) do campo magnético

H ao longo da: (a) curva aberta determinada, num plano z=z

(b) curva fechada determinada, nesse plano, pela superfície

pelas superfícies ρ=a, ρ=b, φ=φ1 e φ

3- (2,5) Num sistema de coordenadas cilíndricas, um vetor indução magnética é dado por

B(0≤ρ≤a,t)=Bosenωt(z/z), onde Bo,

Obtenha a expressão do campo elétrico em qualquer ponto dessa região e em

4-(parte A)-(1,0) Um fluxo elétrico uniforme aponta para fora do papel em uma região circular de raio R. O fluxo

elétrico total através da região é ΦE

magnético induzido a uma distância radial

(parte B)-(1,5) Quando empurramos um imã na direção de uma espira (figura), o agente que causa o movimento

do imã sofrerá sempre a ação de uma força resistente, o que o obrigará a realização de um trabalho a fim de

conseguir efetuar o movimento desejado. A) explique o aparecimento desta força resistente. B) Se cortarmos a

espira como mostra a figura abaixo, será necessário realizar trabalho para movimentar o imã? haverá uma

corrente induzida na espira ? haverá uma f.e.m induzi

5-(2,5) A amplitude do campo elétrico associado a uma onda luminosa harmônica, plana e polarizada linearmente,

é: Ez=(2 V/m)cos[1015π(t-x/0,65c)] no interior de um vidro, onde c é a velocidade da luz no vácuo, t é expresso em

segundos e x em metros. Determine: A) (0,2)a frequência

velocidade de propagação no meio; D) (0,2) o índic

vetor de Poyinting instantâneo;G) (0,5) a intensidade da onda;

Tópicos de Eletromagnetismo I – Prof. Antônio Carlos

Universidade Federal do Rio de Janeiro Instituto de Física

Curso de Licenciatura em Física Segunda Chamada de Tópicos de Eletromagnetismo I (2012/1)

Prof. Antônio Carlos ([email protected])

Caro aluno, você deve escolher quatro questões para resolver. Boa sorte!

(2,5) A densidade volumar de uma distribuição de cargas é, em coordenadas esféricas ρ (0≤a)=0, onde a, b e ρo são positivos.

gráfico da densidade contra r;

Determine o raio b que anula a carga total da distribuição;

D nas quatro regiões da distribuição, quando b tiver o valor determinado no

Faça um gráfico da componente radial de D contra r;

(2,5) Uma corrente constante, retilínea e ilimitada I flui ao longo do eixo z de um sistema de coordenadas

cilíndricas, no seu sentido positivo. Calcule explicitamente a integral curvilínea (de caminho) do campo magnético

rta determinada, num plano z=zo, pela superfície ρ=a, entre os ângulos

(b) curva fechada determinada, nesse plano, pela superfície ρ=a; (c) curva fechada determinada, nesse plano,

φ=φ2.

de coordenadas cilíndricas, um vetor indução magnética é dado por

, ω e a são uniformes, constantes e positivos e (z/z) é o unitário na direção z.

btenha a expressão do campo elétrico em qualquer ponto dessa região e em qualquer instante.

(1,0) Um fluxo elétrico uniforme aponta para fora do papel em uma região circular de raio R. O fluxo

E=Kt, onte K é uma constante e t é o tempo. Determine o módulo do campo

ido a uma distância radial ρ.

Quando empurramos um imã na direção de uma espira (figura), o agente que causa o movimento

do imã sofrerá sempre a ação de uma força resistente, o que o obrigará a realização de um trabalho a fim de

efetuar o movimento desejado. A) explique o aparecimento desta força resistente. B) Se cortarmos a

espira como mostra a figura abaixo, será necessário realizar trabalho para movimentar o imã? haverá uma

corrente induzida na espira ? haverá uma f.e.m induzida?

(2,5) A amplitude do campo elétrico associado a uma onda luminosa harmônica, plana e polarizada linearmente,

x/0,65c)] no interior de um vidro, onde c é a velocidade da luz no vácuo, t é expresso em

segundos e x em metros. Determine: A) (0,2)a frequência ω da luz; B) (0,2)o seu comprimento de onda; C) (0,2) a

velocidade de propagação no meio; D) (0,2) o índice de refração do vidro; E) (0,5) o campo magnético; F) (0,5) O

vetor de Poyinting instantâneo;G) (0,5) a intensidade da onda;

81

(2012/1)

questões para resolver. Boa sorte!

≤ r<a)=0, ρ (a≤

nas quatro regiões da distribuição, quando b tiver o valor determinado no

(2,5) Uma corrente constante, retilínea e ilimitada I flui ao longo do eixo z de um sistema de coordenadas

cilíndricas, no seu sentido positivo. Calcule explicitamente a integral curvilínea (de caminho) do campo magnético

=a, entre os ângulos φ=φ1 e φ=φ2;

=a; (c) curva fechada determinada, nesse plano,

de coordenadas cilíndricas, um vetor indução magnética é dado por

é o unitário na direção z.

qualquer instante.

(1,0) Um fluxo elétrico uniforme aponta para fora do papel em uma região circular de raio R. O fluxo

=Kt, onte K é uma constante e t é o tempo. Determine o módulo do campo

Quando empurramos um imã na direção de uma espira (figura), o agente que causa o movimento

do imã sofrerá sempre a ação de uma força resistente, o que o obrigará a realização de um trabalho a fim de

efetuar o movimento desejado. A) explique o aparecimento desta força resistente. B) Se cortarmos a

espira como mostra a figura abaixo, será necessário realizar trabalho para movimentar o imã? haverá uma

(2,5) A amplitude do campo elétrico associado a uma onda luminosa harmônica, plana e polarizada linearmente,

x/0,65c)] no interior de um vidro, onde c é a velocidade da luz no vácuo, t é expresso em

da luz; B) (0,2)o seu comprimento de onda; C) (0,2) a

e de refração do vidro; E) (0,5) o campo magnético; F) (0,5) O

Page 82: Topicos de Eletromagnetismo I

82 FIW 591 – Tópicos de Eletromagnetismo I – Prof. Antônio Carlos

Gabarito: 1-(b) b=(9/2)1/3

a; (c) D (0≤r<a) = D (r>2a) =0; D (a≤r<b) =(ρo/3)(8 a3r

-3-1)r; 2- a)Γ=I(φ2 -φ1)/2π;

b) Γ=I; c) Γ=0; 3- E(0≤ρ≤a,t)=-(1/2)ωρBocosωt na direção do unitário tangencial, 5)

Page 83: Topicos de Eletromagnetismo I

FIW 591 – Tópicos de Eletromagnetismo I

Atenção! Duas respostas erradas elimina uma certa!

1- (0,5) Considere um plano (infinito) com uma densidade de carga constante (estacionária e uniforme). Na figura, estão

representadas quatro superfícies fechadas S

plano carregado. Dentre elas, qual(is) exatamente aque

expressão geral para o campo elétrico num ponto genérico, fora do plano, a partir da lei de Gauss?

a) ( ) S1;

b) ( ) S2;

c) ( ) S3;

d) ( ) S4;

e) ( ) S1 e S2;

f) ( ) S2 e S3;

g) ( ) S2 e S4 ;

h) ( ) S3 e S4 ;

2- (0,5) Um campo eletrostático possui superfícies equipotenciais planas, paralelas, como mostrado na figura, numa vista

de perfil, pelas três retas tracejadas, igualmente espaçadas de uma distância L, com V

mostradas quatro trajetórias

equipotenciais. Considere as afirmações: (I) o vetor campo elétrico médio E

–(V2/L)j ; (II) o trabalho realizado pela força

as trajetórias mostradas; (III) o trabalho realizado realizado pela força eletrostática ao deslocar

carregada na trajetória de g para h é negativo. Qual(is) de tais af

A) ( ) nenhuma;

B) ( ) I;

C) ( ) II;

D) ( ) III;

E) ( ) I e II;

F) ( ) I e III;

G) ( ) II e III;

H) ( ) todas;

3- (0,5) Quatro cargas pontuais q,

respectivamente. Em termos da delta de Dirac, a densidade volumar de cargas desta distribuição é dada por:

a) ( ) ρ=q[δ(x+1)δ(y)δ(z)-

b) ( ) ρ=q[δ(x-1)δ(y)δ(z)+

c) ( ) ρ=q[δ(x+1)δ(y)δ(z)+

d) ( ) ρ=q[δ(x-1)δ(y)δ(z)-

e) ( ) ρ=q[δ(x+1)δ(y)δ(z)-

4- (0,5) Considere duas esferas condutoras isoladas cada uma tendo uma carga Q. As esferas tem raios a e b, onde b>a.

Qual esfera tem maior potencial (tomando o infinito como referência) e qual esfera tem um campo elétrico

intenso na superfície?

A) ( ) VA > VB; E

B) ( ) VA > VB; E

C) ( ) VA > VB; E

D) ( ) VA = VB; E

E) ( ) VA = VB; E

Tópicos de Eletromagnetismo I – Prof. Antônio Carlos

Universidade Federal do Rio de Janeiro Instituto de Física

Curso de Licenciatura em Física Primeira Prova de Tópicos de Eletromagnetismo I (2012/2)

Prof. Antônio Carlos ([email protected])

Atenção! Duas respostas erradas elimina uma certa!

Considere um plano (infinito) com uma densidade de carga constante (estacionária e uniforme). Na figura, estão

representadas quatro superfícies fechadas Si (i = 1, 2, 3, 4), com disposições particulares simétricas com respeito ao

e elas, qual(is) exatamente aquela(s) que é (são) apropriada(s) para a determinação de uma

ampo elétrico num ponto genérico, fora do plano, a partir da lei de Gauss?

Um campo eletrostático possui superfícies equipotenciais planas, paralelas, como mostrado na figura, numa vista

de perfil, pelas três retas tracejadas, igualmente espaçadas de uma distância L, com V1 =2V

mostradas quatro trajetórias orientadas, por curvas contínuas, que partem da equipotencial V1

equipotenciais. Considere as afirmações: (I) o vetor campo elétrico médio E12 entre as equipotenciais V

; (II) o trabalho realizado pela força eletrostática ao deslocar-se uma partícula carregada é o mesmo em todas

as trajetórias mostradas; (III) o trabalho realizado realizado pela força eletrostática ao deslocar

carregada na trajetória de g para h é negativo. Qual(is) de tais afirmativas está(ão) correta(s)?

( ) nenhuma;

(0,5) Quatro cargas pontuais q, -q, 2q e -2q estão localizadas nas posições (-1,0,0), (0,-1,0), (0,0,

respectivamente. Em termos da delta de Dirac, a densidade volumar de cargas desta distribuição é dada por:

- δ(x)δ(y+1)δ(z)-2δ(x)δ(y)δ(z+1)-2δ(x+1)δ(y+1)δ(z+1)]

(z)+ δ(x)δ(y-1)δ(z)+2δ(x)δ(y)δ(z-1)+2δ(x-1)δ(y-1)δ(z-1)]

(z)+ δ(x)δ(y+1)δ(z)+2δ(x)δ(y)δ(z+1)+2δ(x+1)δ(y+1)δ(z+1)]

- δ(x)δ(y-1)δ(z)+2δ(x)δ(y)δ(z-1)-2δ(x-1)δ(y-1)δ(z-1)]

- δ(x)δ(y+1)δ(z)+2δ(x)δ(y)δ(z+1)-2δ(x+1)δ(y+1)δ(z+1)]

(0,5) Considere duas esferas condutoras isoladas cada uma tendo uma carga Q. As esferas tem raios a e b, onde b>a.

Qual esfera tem maior potencial (tomando o infinito como referência) e qual esfera tem um campo elétrico

; EA > EB;

; EA = EB;

; EA < EB;

; EA = EB;

; EA > EB;

F) ( ) VA =VB; EA < E

G) ( ) VA < VB; EA = E

H) ( ) VA < VB; EA < E

I) ( ) VA < VB; EA > E

83

(2012/2)

Considere um plano (infinito) com uma densidade de carga constante (estacionária e uniforme). Na figura, estão

(i = 1, 2, 3, 4), com disposições particulares simétricas com respeito ao

la(s) que é (são) apropriada(s) para a determinação de uma

ampo elétrico num ponto genérico, fora do plano, a partir da lei de Gauss?

Um campo eletrostático possui superfícies equipotenciais planas, paralelas, como mostrado na figura, numa vista

=2V2=3V3. Além disso, são

1 e passam pelas demais

entre as equipotenciais V1 e V2 é dado por

se uma partícula carregada é o mesmo em todas

as trajetórias mostradas; (III) o trabalho realizado realizado pela força eletrostática ao deslocar-se uma partícula

1,0), (0,0,-1) e (-1,-1,-1),

respectivamente. Em termos da delta de Dirac, a densidade volumar de cargas desta distribuição é dada por:

(0,5) Considere duas esferas condutoras isoladas cada uma tendo uma carga Q. As esferas tem raios a e b, onde b>a.

Qual esfera tem maior potencial (tomando o infinito como referência) e qual esfera tem um campo elétrico mais

< EB;

= EB;

< EB;

> EB;

Page 84: Topicos de Eletromagnetismo I

84 FIW 591 – Tópicos de Eletromagnetismo I – Prof. Antônio Carlos

5- (0,5)Considere uma distribuição de cargas da figura. São oito segmentos retilíneos de mesmo comprimento,

uniformemente carregados com densidade linear de mesmo módulo λ > 0. O ângulo entre segmentos vizinhos é o

mesmo (45O). Qual das alternativas melhor representa o campo elétrico resultante em O?

6- (0,5) O quadrado e o círculo na figura abaixo estão imersos em um mesmo campo elétrico uniforme. O diâmetro do

círculo é igual ao lado do quadrado. A razão entre o fluxo do campo elétrico através do quadrado e do círculo

(Φquadrado/Φcírculo)é:

A) ( ) 1/π;

B) ( ) 2/π;

C) ( ) 1;

D) ( ) 4/π;

E) ( ) π;

F) ( ) π/2;

G) ( ) π/4;

7- (0,5) Uma partícula puntiforme carregada é colocada no centro de uma superfície gaussiana esférica S. Afirma-se que o

fluxo do campo elétrico em S será alterado se: i) a superfície S for substituída pela superfície de um cubo de volume

diferente diferente da esfera mas com o mesmo centro. ii) Se a partícula for arrastada do centro da superfície original

mas ainda continuando dentro da superfície S. iii) A carga for removida para fora da superfície S. iv) Uma segunda carga

for colocada próxima e fora da superfície S. v) Uma segunda carga for colocada dentro da superfície S. Qual(is) das

afirmações acima estão corretas:

A) ( ) todas elas;

B) ( ) nenhuma delas;

C) ( ) somente i) e ii);

D) ( ) somente iii) e iv);

E) ( ) somente i) e iii);

F) ( ) somente ii) e iii);

G) ( ) somente iii) e v);

H) ( ) somente iv) e v);

8- (0,5)Um cilindro feito de material isolante é colocado em um campo elétrico externo, conforme mostrado na figura. O

fluxo do campo elétrico através da sua face direita, da sua face esquerda e sobre toda a sua superfície é,

respectivamente:

A) ( ) nulo, nulo, nulo;

B) ( ) nulo, nulo, positivo;

C) ( ) nulo, positivo, nulo;

D) ( ) nulo, positivo, positivo;

E) ( ) nulo, positivo, negativo;

F) ( ) nulo, negativo, negativo;

G) ( ) positivo, negativo, positivo;

H) ( ) negativo, negativo, negativo;

I) ( ) positivo, positivo, positivo;

J) ( ) negativo, negativo, nulo;

K) ( ) negativo, positivo, positivo;

L) ( ) negativo, positivo, nulo;

M) ( ) positivo, nulo, nulo;

N) ( ) positivo, positivo, nulo;

O) ( ) positivo, negativo, nulo;

Page 85: Topicos de Eletromagnetismo I

85 FIW 591 – Tópicos de Eletromagnetismo I – Prof. Antônio Carlos

9- (2,0) a) (1,0) O potencial elétrico é dado pela expressão 222)( zyxrV ++= calcule o vetor campo elétrico.

b) (1,0) Dado o campo elétrico E=xi+yj+zk no vácuo, calcule a densidade volumar de carga ρ que dá origem a este

campo em função de εo.

10- (4,0) A Fig. 1 mostra uma placa fina e muito grande

que possui uma densidade superficial e carga

constante σ. A placa é recoberta lateralmente por

duas lâminas de espessura D e densidade volumar

de carga constante ρ. (a) (2,0) Utilizando a lei de

Gauss, obtenha o vetor campo elétrico E(z)

produzido pela distribuição de cargas a uma

distância |z| da placa central para os casos em que:

(i) –D ≤ z ≤D e (ii) z ≤ -D. Faça uma gráfico

esboçando o comportamento da componente Ez

versus z, no intervalo z∈(-2D,2D), para o caso em

que σ e ρ são positivos (1,7 ponto). (b) (2,0) Usando

a expressão para o vetor E(z) e tomando como

referência o potencial elétrico VD ≡ V(z=D) na

superfície externa da lâmina lateral (à direita),

obtenha a expressão para o potencial elétrico V(z)

produzido pela distribuição de cargas a uma

distância |z| considerando os mesmos casos acima,

ou seja, em que: (i) –D ≤ z ≤D e (ii) z ≤ -D. Faça um

gráfico esboçando o comportamento de V versus z,

no intervalo z∈(-2D,2D), para o caso em que σ e ρ

são positivos.

Page 86: Topicos de Eletromagnetismo I

FIW 591 – Tópicos de Eletromagnetismo I

Nome:___________________________________________________________________

1- (2,5) Duas cascas esféricas de metal concêntricas de raios

condutividade elétrica σA) Determine o campo elétrico com função do tempo nas regiões r < a , a < r < b e r>b.

B) Qual a corrente elétrica total

C) Calcule a potência dissipada por unidade de volume no material devido à passagem de corrente como

função do tempo. Mostre que a energia total dissipada é igual à energia e

2- (2,5) Uma longa casca cilíndrica tem raio interno a, raio externo b e conduz corrente I paralela ao eixo central.

Considere que, no interior do material da casca, a densidade de corrente está uniformemente distribuída

Determine uma expressão para a

3- (2,5) Um fio infinitamente longo está ao longo do eixo z e conduz uma corrente de I na direção +z. Um segundo

fio infinitamente longo é paralelo ao eixo z intercepta o eixo x em x=x

se o campo magnético é zero em (x

4- (2,5) Um longo solenoide tem n voltas por unidade de comprimento e conduz uma corrente que varia com o

tempo de acordo com I=

induzido em pontos próximos ao plano equidistante das extremidades do solenoide como função do tempo t e da

distância perpendicular r do eixo do solenoide para (a) r<

5- (2,5) Esta questão consiste de cinco subquestões, cada uma valendo 0,5 ponto.

respostas acompanhadas das respectivas justificativas.

5.1 – Um fio transporta uma corrente I ao longo do eixo z

A) ( ) H=(I/2πa)j em (x=a,y=0,z=0);

B) ( ) H=-(I/2πa)i em (x=0,y=a,z=a);

C) ( ) H=-(I/2πa)j em (x=

D) ( ) H=(I/2πa)i em (x=0,y=

E) ( ) H=(I/2πa)j em (x=a,y=a,z=0);

5.2- um fio circular de raio a no plano xy

magnético deste fio é:

A) ( ) m=Iπak

B) ( ) m=-I2πak

C) ( ) m=I2πak

D) ( ) m=-Iπa2k

E) ( ) m=Iπa2k

5.3-Se m é módulo do momento de dipolo magnético, o

ponto (x=0,y=0,z=a) é:

A) ( ) A=(µo/4π)m/a2k

B) ( ) A=-(µo/4π)m/a2k

C) ( ) A=0

D) ( ) A=(µo/4π)m/ak

E) ( ) A=-(µo/4π)m/ak

5.4 – O fluxo do campo magnético através de um anel

é dado em segundos e o fluxo em webers. A f.e.m induzida no anel e a corrente elétrica que circula em t=0 s é:

A) ( ) ε =1 V e I = 1 A;

B) ( ) ε =-1 V e I = 1 A;

C) ( ) ε =0 V e I = 0 A;

D) ( ) ε =1 V e I = 0 A;

E) ( ) ε =0 V e I = 1 A;

5.5- A força resultante em um fio circular de raio a, no plano xy r que transporta uma corrente I em uma região de

um campo magnético uniforme

A) ( )2πaBI;

B) ( )πa2BI;

C) ( )πaBI;

D) ( )2aBI;

E) ( ) zero;

Tópicos de Eletromagnetismo I – Prof. Antônio Carlos

Universidade Federal do Rio de Janeiro Instituto de Física

Curso de Licenciatura em Física Segunda Prova de Tópicos de Eletromagnetismo I (2012/2)

Prof. Antônio Carlos ([email protected])

Nome:___________________________________________________________________

Escolha quatro questões para resolver. Boa sorte!

(2,5) Duas cascas esféricas de metal concêntricas de raios a e b (b > a) estão separadas por um material de

σ. No instante t=0 a casca esférica interna de raio a possui uma carga Q.

Determine o campo elétrico com função do tempo nas regiões r < a , a < r < b e r>b.

Qual a corrente elétrica total que flui no material entre as esferas em função do tempo?

Calcule a potência dissipada por unidade de volume no material devido à passagem de corrente como

função do tempo. Mostre que a energia total dissipada é igual à energia eletrostática no material em

(2,5) Uma longa casca cilíndrica tem raio interno a, raio externo b e conduz corrente I paralela ao eixo central.

Considere que, no interior do material da casca, a densidade de corrente está uniformemente distribuída

Determine uma expressão para a magnitude do campo magnético para (a) 0<r<a, (b) a<r<b e (c) r>b.

Um fio infinitamente longo está ao longo do eixo z e conduz uma corrente de I na direção +z. Um segundo

fio infinitamente longo é paralelo ao eixo z intercepta o eixo x em x=xo. (a) Determine a corrente no segundo fio

se o campo magnético é zero em (xo/2, 0,0). (b) Qual é o campo magnético (vetor) em (2xo

(2,5) Um longo solenoide tem n voltas por unidade de comprimento e conduz uma corrente que varia com o

tempo de acordo com I=Iosenωt. O solenoide tem seção transversal circular de raio R. Determine o campo elétrico

induzido em pontos próximos ao plano equidistante das extremidades do solenoide como função do tempo t e da

distância perpendicular r do eixo do solenoide para (a) r<R e (b) r>R.

(2,5) Esta questão consiste de cinco subquestões, cada uma valendo 0,5 ponto. Apenas serão consideradas as

respostas acompanhadas das respectivas justificativas.

Um fio transporta uma corrente I ao longo do eixo z (no sentido +z). Qual a opção incorreta?

em (x=a,y=0,z=0);

em (x=0,y=a,z=a);

em (x=-a,y=0,z=-a);

em (x=0,y=-a,z=2a);

em (x=a,y=a,z=0);

um fio circular de raio a no plano xy transporta uma corrente I no sentido horário. O momento de dipolo

Se m é módulo do momento de dipolo magnético, o potencial vetor do fio circular da questão anterior

O fluxo do campo magnético através de um anel metálico de resistência R=1,0 Ω é dado por

é dado em segundos e o fluxo em webers. A f.e.m induzida no anel e a corrente elétrica que circula em t=0 s é:

A força resultante em um fio circular de raio a, no plano xy r que transporta uma corrente I em uma região de

um campo magnético uniforme B=Bok é:

86

Eletromagnetismo I (2012/2)

Nome:___________________________________________________________________

) estão separadas por um material de

. No instante t=0 a casca esférica interna de raio a possui uma carga Q.

que flui no material entre as esferas em função do tempo?

Calcule a potência dissipada por unidade de volume no material devido à passagem de corrente como

letrostática no material em t=0.

(2,5) Uma longa casca cilíndrica tem raio interno a, raio externo b e conduz corrente I paralela ao eixo central.

Considere que, no interior do material da casca, a densidade de corrente está uniformemente distribuída.

magnitude do campo magnético para (a) 0<r<a, (b) a<r<b e (c) r>b.

Um fio infinitamente longo está ao longo do eixo z e conduz uma corrente de I na direção +z. Um segundo

Determine a corrente no segundo fio

o,0,0)?

(2,5) Um longo solenoide tem n voltas por unidade de comprimento e conduz uma corrente que varia com o

t. O solenoide tem seção transversal circular de raio R. Determine o campo elétrico

induzido em pontos próximos ao plano equidistante das extremidades do solenoide como função do tempo t e da

Apenas serão consideradas as

opção incorreta?

transporta uma corrente I no sentido horário. O momento de dipolo

potencial vetor do fio circular da questão anterior no

é dado por Φ=t2 –t , onde t

é dado em segundos e o fluxo em webers. A f.e.m induzida no anel e a corrente elétrica que circula em t=0 s é:

A força resultante em um fio circular de raio a, no plano xy r que transporta uma corrente I em uma região de